Vous êtes sur la page 1sur 102

Strategy Thinking:

Question 1

Tilley is on a committee of volunteers judging an essay contest for high school students. The contest is aimed at
encouraging students to consider a career in corporate management and this year the theme is The Importance of
Strategic Thinking. Tilley reads four of the essays and rejects three of them because they have errors. Which of the
following essay excerpts was correct?

Correct

Good choice. You answered: "Strategic thinking involves thinking about how managers interact with other
departments."

Every manager in your organization has a unique view of how the company operates. By asking peer managers
questions about how they interact with people from various parts of the enterprise, you strengthen your
understanding of how your actions might affect them and their work.

REVIEW MATERIAL
Learn > Strategic Thinking Overview > The power of collaboration

Question 2

You are on a hiring committee choosing a new director of product development. Senior managers have made it
clear that above all, they want to hire a strategic thinker for the job. The committee has asked the four finalists to
write a brief description of their strengths as managers. Which of the following candidates would be the best choice
for the job?

Correct

Good choice. You answered: Mary wrote, "I research situations carefully, but when the solution seems to be
unclear, I'll take a chance and play my hunches."

Strategic thinkers factor hunches into their decision making without allowing their hunches to dominate the final
outcome. They also objectively analyze situations, grasp abstract ideas, and generate a wide range of options.

REVIEW MATERIAL
Learn > Defining Characteristics of Strategic Thinkers > Cognitive capacities

Question 3

Armand is an executive at a large broadcasting company. He has just been put in charge of repositioning the
companys brand and he wants to make sure he brings strategic thinking to the task. Which of the following would
he likely not include as part of his strategic-thinking process?
Incorrect

Not the correct choice. You answered: Make a list of information needed and arrange it in order of importance.

Sorting out and prioritizing the most important information is part of the strategic-thinking process.

The correct choice is: Ensure that all long-term goals take precedence over short-term concerns.

It is important to recognize the advantages and disadvantages of an idea or course of action, making choices that
balance short-term and long-term concerns.

REVIEW MATERIAL
Learn > Strategic Thinking as a Process > Phase 2: Applying your skills

Question 4

Marsha is in charge of developing new products for a frozen foods company. To get a better idea of what might suit
the marketplace, she decides to assess developments outside of her company. Which of the following questions
would she be least likely to ask in this regard?

Correct

Good choice. You answered: What does the allocation of our resources say about our higher-level strategy?

Asking how senior executives allocate resources is appropriate when trying to understand your company's and
unit's strategy, not when trying to assess developments outside of your company.

REVIEW MATERIAL
Learn > Seeing the Big Picture > Understand your company's and unit's strategies

Question 5

At a seminar on business planning, the facilitator asked participants to write down one objective each. He then
chose four managers to read their objectives aloud. The participants were a bit offended when he told them only
one person had written a SMART objective until he explained what that meant. Which of the four statements was
a SMART objective?

Correct

Good choice. You answered: "My unit will increase sales of washing machines by at least 5 percent by the end of
the next quarter."

The objective of increasing sales of washing machines by at least 5 percent by the end of the next quarter is
specific (an increase in washing machine sales), measurable sales will increase 5 percent), achievable (an increase
in sales is likely to be achievable in any industry), realistic (the size of the increase is not too large), and time
bound (it will be achieved by the end of the next quarter).
REVIEW MATERIAL
Learn > Clarifying Strategic Objectives > Developing objectives

Question 6

The medical devices company where John works is having trouble maintaining quality control. As production
manager, John feels he will have to think strategically to solve the problem. He asks for advice from his father who
worked in the medical devices field for 30 years. What is his father least likely to tell John?

Incorrect

Not the correct choice. You answered: Get to know the companys organization chart.

Solutions cannot be made in a vacuum. The companys organizational chart can help John find out what major
functions other groups in the company are responsible for and how these groups affect his groups efforts and vice
versa.

The correct choice is: Try to focus on the local context of the problems rather than on their impacts across the
organization.

A hallmark of strategic thinking is the capacity to understand relationships across different parts of your
organization and to spot patterns and trends in seemingly unrelated events and information.

REVIEW MATERIAL
Learn > Identifying Relationships, Patterns, and Trends > Make connections

Question 7

Creative thinking is the lifeblood of the design firm Studio 3 Solutions. As a result, the owner likes to have all new
managers sit in on a video presentation about the role of creative thinking in management. What advice is most
likely to be included in the video?

Correct

Good choice. You answered: Don't be afraid to ask people who don't work directly for you for ideas.

Deliberately inviting people who work in other parts of the organization to share their views of a problem or
challenge can help you see that there is more than one way to perceive a situation.

REVIEW MATERIAL
Learn > Thinking Creatively > Gather others' perspectives

Question 8

Triangle Machinery recently started receiving a larger than usual number of product returns from customers. Jamilla
has been assigned to find the source of the problem. Her boss tells her to begin by gathering information. What
other advice is he most likely to give her?
Correct

Good choice. You answered: Ignore information that is going to be very time-consuming to collect, even if it seems
like something youd like to know.

Don't bother considering information that's unnecessary, marginally useful, or overly time-consuming to collect
even if it strikes you as interesting. Focus your information-gathering efforts on data that will help you move
forward to a resolution.

REVIEW MATERIAL
Learn > Analyzing Information > Avoid irrelevant information

Question 9

Theres a mountain of work on Sybils desk. Shes in charge of launching a new Internet shopping service for her
employer and shes beginning to wonder how to prioritize all the things shes trying to get done. That night, she
calls a colleague for advice. What did the colleague most likely tell her?

Correct

Good choice. You answered: "Put your tasks in order according to how well they support the goals of the
corporation."

Prioritize your list of items based on how well each one supports your company's and unit's high-level goals. Ask
yourself, "Which of these items are the most critical that is, which will generate the most important results for my
group and company?"

REVIEW MATERIAL
Learn > Prioritizing Your Actions > Keep your eye on the big picture

Question 10

As part of a company-wide program, Kelly is under pressure to cut costs in her regional office. However, her office is
one of the few that is still growing its business and that means she faces trade-offs in areas such as customer
service and overhead expenses. What would be the best advice to help her deal with such trade-offs?

Correct

Good choice. You answered: Take a stand and make it clear what you are not willing to do.
Strategy Execution:-

Question 1

Which of the following statements about strategy is false? An organizations strategy ...

Correct

Good choice. You answered: Achieves the best results when planned solely by senior management.

It is not true that the best results are achieved when senior managers plan the strategy on their own. Organizations
that fail to include units in the strategic planning process typically receive results inferior to those that do. By
undertaking the process together, senior management and units ensure that a companys strategies are tightly
aligned and that successful implementation can follow.

REVIEW MATERIAL
Learn > Strategy Overview > How is strategy developed and who is involved?

Question 2

Jade Textiles formulates a strategic plan to refocus its market approach. The plan has three parts: The first part
summarizes the companys mission, vision, and competitive advantages. The second part enables the company to
measure how it is performing in key results areas. The third part outlines weaknesses to be addressed and
opportunities to be seized. What important element, usually included in a strategic plan, is missing?

Correct

Good choice. You answered: Action plans.

Jades strategic plan was missing an action plan the specific steps the company needs to take to accomplish its
priority issues and thereby achieve its objectives. A high-level action plan typically includes: a description of the
priority issue and its importance, objectives, key steps, resources and interlocks required, and anticipated cost and
gain.

REVIEW MATERIAL
Learn > The Strategic Plan > Elements
Question 3

Galaxy Sound Inc. has begun a strategic planning process to reposition itself globally as a leader in high-end home
entertainment systems. Senior management has delegated strategic objectives and priority issues to units. The
units, in turn, have started to analyze internal and external trends and forces. What trends and forces will be of
primary importance to units when they analyze their external environment?

Correct

Good choice. You answered: Market developments, such as competitive advantages.

In the external analysis of trends and forces, the examination of market segmentation and customer needs is of
primary importance. For this analysis, a unit researches market segments groups of customers within a broad
market whose needs and wants are similar. The analysis asks questions pertaining to current and future
segmentation, worthwhile targets, and gaps the company must fill to beat the competition.

REVIEW MATERIAL
Learn > The Strategic Planning Process > Analyze external and internal factors

Question 4

Newtown Health Supplies is conducting a SWOT analysis as a part of their strategic planning process. The analysis
reveals that the senior citizen population in their community is declining rapidly. In the SWOT analysis, this trend
would be a classified as a ...

Correct

Good choice. You answered: Threat.

The declining senior citizen population would be considered a threat to Newtown. A threat is an event or force,
outside of Newtowns control, that the company must plan for. In this case, since the company makes health
supplies, a reduction in a customer groups population could mean lost sales in the future.

REVIEW MATERIAL
Learn > The Strategic Planning Process > Perform SWOT analyses

Question 5

Your company is engaged in a strategic planning exercise. Senior management has identified several market trends,
issues and core internal processes that are most relevant to the companys vision of continued expansion. A SWOT
analysis has revealed external events that may provide the company with opportunities to fulfill that vision. What is
your next step?

Correct

Good choice. You answered: Drafting priority issues.


The strategic planning process has five steps: analyzing external and internal factors, conducting SWOT analyses,
drafting priority issues, developing high-level action plans, and finalizing the strategic plan. Thus the next step after
your internal/external forces and SWOT analyses involves drafting priority issues broad areas in which the
company should focus its efforts for the long term.

REVIEW MATERIAL
Learn > The Strategic Planning Process > Draft priority issues

Question 6

Which statement about the strategic planning process is true?

Correct

Good choice. You answered: Strategic planning is not a linear sequence of activities.

This statement is true. Strategic planning is an iterative process that takes time and requires a series of back-and-
forth communications between senior management and units, whereby all parties examine, discuss, and refine the
plan. As a result, various planning streams often happen in parallel.

REVIEW MATERIAL
Learn > The Strategic Planning Process > Process overview

Question 7

Danielle is leading a team responsible for creating a strategic initiative action plan for her unit at J&L Office
Products. The priority issue is to open mini-stores in new industrial complexes to capture more of the suburban
business market and increase revenue. When Danielle develops the impact estimate portion of the action plan,
what will she describe?

Correct

Good choice. You answered: The expected costs of building the stores and the anticipated revenue.

The impact estimate part of the plan describes the anticipated costs associated with building the mini-stores (e.g.,
expense capital, capital, equipment, etc.) and the revenue potential of the project over a specific period of time.

REVIEW MATERIAL
Learn > Strategic Initiative Action Plan > Components

Question 8

Which of the following is a not-so-SMART objective?

Correct
Good choice. You answered: Increase the number of Quickstar 400SX laser printers produced by the end of the
first quarter.

The objective set for the production of Quickstar printers is not-so-SMART. It is not specific it fails to specify how
much the number of printers will increase by the end of the first quarter. A SMART objective is specific, measurable,
achievable, realistic, and time bound.

REVIEW MATERIAL
Learn > Defining Objectives and Metrics > Write objectives

Question 9

At Creston Surgical Supplies, Claytons manufacturing unit has put a priority on using robotics to produce surgical
instruments. Early on, when Clayton creates his action plan, he asks the engineering department to help specify
appropriate robotics technology. However, later when Clayton begins to implement his plan, the head of
engineering is unwilling to help. He tells Clayton that his department is not obliged to help out and does not have
the resources to do so. As a result, Claytons action plan is stalled. What could Clayton have done to avoid this
problem? He could have ...

Correct

Good choice. You answered: Confirmed and documented interlocks.

Clayton might have avoided the problem with engineering if he had documented his interlock agreement his
expectations for the engineering department in specifying technology before implementing his plan. To ensure
accountability, it is wise for managers to document all interlock needs, expectations, and obligations as well as
any changes in the interlocks. Failure to agree on interlock arrangements is a source of great conflict between
groups in organizations especially during times of tight resources.

REVIEW MATERIAL
Learn > Clarifying Interlocks > Collaborate cross-functionally

Question 10

Kim is preparing a quarterly review for the strategic market expansion initiative for Dixie Dream fast-food outlets.
What is the chief purpose of this review?

Correct

Good choice. You answered: To assess her units progress and alignment on its action plan.

Quarterly reviews are one- to two-page reports to senior management created for each action plan a unit is
implementing. The reviews are an important tool for assessing progress and checking alignment. They describe:
what the unit has accomplished, what the unit planned to accomplish but didnt, key issues that need resolution,
decisions or resources required, and performance to objectives.

REVIEW MATERIAL
Learn > Keeping Action Plans Aligned and on Course > Review progress
Question 11

Jake is head of a shipment tracking division of a courier company and is working on a strategic initiative to
streamline deliveries to Asia. In the quarterly review of his action plan, he notes the following: Changes in work
process and political blockages are causing delays, and actual costs are running higher than originally estimated.
These statements reflect...

Incorrect

Not the correct choice. You answered: A lack of strategic mindset.

A strategic mindset is the shared belief that strategy is everyones job.

The correct choice is: Sources of misalignment.

Changes in work process, political interferences, and inaccurate estimates are examples of things that can cause
misalignment and derail an action plan. Even the most carefully thought out action plans can fall victim to
misalignment or become derailed. Whatever their cause, misalignment and derailment are key issues that need to
be aired and addressed during quarterly reviews.

REVIEW MATERIAL
Learn > Keeping Action Plans Aligned and on Course > Understand misalignment

Question 12

You are the director of the product development group at Cutting Edge Video Games. Your company has a strategic
initiative to enter and dominate the pre-teen video game market. You want to cultivate a strategic mindset to help
your unit excel in this initiative. At a recent staff meeting, you made several remarks about the companys new
direction. Which of the following comments would not help your group develop a strategic mindset?

Incorrect

Not the correct choice. You answered: The goal for our product development group is to launch three new
products targeted to the pre-teen market by the end of the fiscal year.

This comment would actually help your group develop a strategic mindset because it explains how your unit will
carry out the companys strategy.

The correct choice is: The marketing group has just completed extensive research on product design. I will
summarize their findings and e-mail the full report by the end of the week.

Discussing marketings research on product design would not help your unit develop a strategic mindset because
it does not communicate the importance of the companys strategy or your teams role in implementing it.

Through meetings, informal conversations, e-mails, and other communication channels, managers must convey
why a companys strategy is necessary, how it will be implemented, what will happen if units are successful or
unsuccessful in carrying out their plans, and what attitudes and behaviors are expected of team members.

REVIEW MATERIAL
Learn > Creating an Environment for Excellence > Develop a "strategic mindset"

Question 13

You are human resources manager leading a strategic initiative that involves streamlining your accounting
processes. Samuel, one of your direct reports, is resisting the change. He is showing a diminished enthusiasm
toward his job and at times even refuses to perform some of the required tasks. What is the best strategy for
dealing with Samuel?

Correct

Good choice. You answered: Involve Samuel as much as possible in the strategic planning and implementation
processes.

In addition to providing more information about the strategic initiative as well as developing a formal performance
improvement plan, involving resisters in the planning and implementation process can also be very effective. If
Samuel participates in the planning and implementation, he may develop a personal investment in the new
initiative.

REVIEW MATERIAL
Learn > Creating an Environment for Excellence > Address resistance

Question 14

Angelo is evaluating his units performance on its overall objectives. He uses a combination of quantitative and
qualitative criteria to assess his groups success. Which of the following evaluative criteria is quantitative?

Correct

Good choice. You answered: Whether the group raised revenues by the desired amount that year.

Criteria that focus on revenue, cost of goods, market share, and so forth are quantitative (measurable). Quantitative
and qualitative criteria are both important for evaluating an individuals or a groups performance.

REVIEW MATERIAL
Learn > Evaluating and Rewarding Performance > Quantitative criteria

Question 15

To support his companys strategic initiative, David has created a rewards program to compensate his team for
good performance. Over lunch, he shares with you his views of a successful rewards program. Which of Davids
statements about rewards programs do you consider not accurate?

Incorrect

Not the correct choice. You answered: Reward systems can work well based on either individual or group
performance.
It is accurate to say that reward systems can work well based on either individual or group performance.

The correct choice is: Perks such as flexible work schedules, job-sharing arrangements, and personal days
dont constitute effective rewards.

Innovation and Implementation:-

Question 1

You have been asked to conduct a seminar for new employees on implementing innovation in your company. At the
end of the seminar, you ask the participants to write a sentence about innovation on a piece of paper and give it to
you. After reviewing the submissions, you find that many of the participants have written statements that are
inaccurate. Which of the following statements is correct?

Correct

Good choice. You answered: A good champion for a creative idea is critical to its successful implementation.

Research suggests that a committed champion is most critical to the successful implementation of an idea,
especially if the idea represents a radical innovation opportunity or if the need for it is not generally recognized.

REVIEW MATERIAL
Learn > Innovation and Innovators > Types of innovators

Question 2
John Corcoran is a junior engineer at Healthways Medical Devices. He has just come up with a way of lowering the
manufacturing cost of the company's Breathe E-Z Ventilator and decides to write a vision statement to promote the
idea. He first visualizes the process that he would go through to make the idea a reality and jots down his thoughts.
He repeats this exercise several times before writing a one-paragraph vision statement. Finally, he asks a friend in
his department to read it and make suggestions. What part of the process could he have done better? He should
have ...

Correct

Good choice. You answered: Visualized the perfect outcome of his idea instead of the process of getting there.

It is recommended that you close your eyes and imagine a time in the future when the innovation has been
successfully implemented. Focus on the ideal outcome, not on how you got there. Then jot down your thoughts.

REVIEW MATERIAL
Learn > Developing an Inspiring Vision of Your Innovation > Where to begin?

Question 3

Lukas Schultz is a product designer with Pocket Data Corporation. He is planning to make a presentation to his team
on a new product concept early next week. To prepare for the meeting, he writes down a few questions that he
anticipates team members will ask. Which of the following questions is the least likely to be raised with this group
of stakeholders?

Incorrect

Not the correct choice. You answered: How will the new product concept affect the team's status in the
organization?

As intermediaries, Lukas' team members will want to know how the new product concept might affect their power
and status in the organization.

The correct choice is: What will be the return on investment for this new product concept?

Intermediaries tend to focus less on the business details and more on identifying the benefits the idea will have for
them. Lukas' team members will most likely be interested in what the innovation requires from them or how it might
benefit their role or function. They may want to know how it might change their work schedules or their power and
status.

REVIEW MATERIAL
Learn > Identifying Stakeholders > The key parties

Question 4

David Suzuki is looking for a gatekeeper to help him develop his idea for a low-cost health insurance plan for senior
citizens. Which of the following statements best describes the role of a gatekeeper?
Correct

Good choice. You answered: An expert in sales who can give David valuable information about the market and
knowledgeably evaluate his ideas.

A gatekeeper is usually an expert in a functional area or subject, such as R&D, manufacturing, or sales. A
gatekeeper has up-to-date knowledge of his or her field and can be a useful to David as a sounding board and an
information resource.

REVIEW MATERIAL
Learn > Cultivating Your Informal Support Network > Assign key roles

Question 5

A colleague asks for your opinion on how to build internal support for a business innovation that she wants to
implement. Which of the following is poor advice to give her?

Correct

Good choice. You answered: Seek input from upper management before anyone else and then build a support
network of peers and colleagues.

Approaching key senior managers too early is as problematic as waiting too long. Typically, successful innovators
first build a support network of peers and colleagues, and then seek support at higher levels.

REVIEW MATERIAL
Learn > Cultivating Your Informal Support Network > Timing

Question 6

Joe Chancellor has drafted a business case for a new line of vegetable-flavored chewing gum. The business case
includes: the goals of the project, expected benefits, major milestones, detailed cost estimates, potential problems
and risks associated with the project, and research staff and equipment needed. Joe wants to show the business
case to upper management and potential supporters throughout the organization. What would you advise Joe to do
next? Joe should ...

Correct

Good choice. You answered: Make a less-detailed version of the business case for potential supporters in the
organization.

Because the business case serves multiple purposes, Joe may want to consider creating at least two versions. In
addition to his detailed business plan, he may want to consider creating a less-detailed version for use when
presenting his idea to potential supporters. This shorter version should be succinct and should take about ten
minutes to present.

REVIEW MATERIAL
Learn > Building a Business Case > Generate more than one version

Question 7

Maisie Bailey was excited about her idea to put drop boxes for Pickup'N Go Courier into each store of a pharmacy
chain. She knew she would encounter resistance from the courier company's scheduling director, so she met with
him to hear what he had to say. Next, she made a formal presentation to key company executives in order to get
their buy-in. To ensure a consistent message, she repeated the exact same presentation several times, in the same
conference room: once for her direct reports, once for her colleagues, and once for key customers. In each
presentation, she was careful to use everyday language and avoid technical jargon. Maisie also made sure that she
created a concise statement that powerfully conveyed the essential benefits of her idea and differentiated it from
others. What could she have done better? She should have ...

Correct

Good choice. You answered: Used different forums or media to communicate with different audiences.

Maisie should have tailored the medium she used for the different audiences she was addressing. For example, she
might have chosen informal meetings with some stakeholders, such as her peers, while more formal group
presentations might have been more appropriate for department heads and key customers.

REVIEW MATERIAL
Learn > Communicating with Stakeholders > Make an effective presentation

Question 8

An editor at a popular trade journal asks you to write an article about managing resistance to change because of
your strong reputation as an innovator in the field. You agree to write the article. Which of the following would you
most likely include in your article?

Incorrect

Not the correct choice. You answered: Explicit resistance from peers and supervisors is often the most difficult to
manage.

Explicit resistance is actually easier to manage than hidden resistance because it is visible.

The correct choice is: Resistance is not usually a problem in itself; rather, it is a symptom of an underlying issue.

Resistance is usually a symptom of an underlying issue. If you can identify the cause of the resistance, you are in a
better position to directly address it.

REVIEW MATERIAL
Learn > Managing Resistance to New Ideas and Change > Causes of resistance

Question 9
Sandy Kindig is a middle manager with the accounting firm Jorkins & Marley. She is promoting the idea of
reorganizing the company so that each accountant is responsible for a single industry sector, rather than having a
mix of clients from different industries. One senior accountant is resistant to the idea, so Sandy decides to use
facilitation to try and win him over. Which of the following strategies best describes the facilitation technique for
overcoming resistance? Sandy should ...

Correct

Good choice. You answered: Offer to organize training sessions to upgrade the senior accountant's knowledge of
the industry sector that interests him most.

Providing skills training or other forms of transition support is an example of using facilitation to overcome
resistance. Sandy should obtain the additional resources necessary to make the senior accountant feel more
comfortable with the reorganization.

REVIEW MATERIAL
Learn > Managing Resistance to New Ideas and Change > Tactics for overcoming resistance

Question 10

You are a manager at Go Play Toys. Thanks to Karen Mosco in R&D, the company's newest offering, the Bubbleup
Musical Bubble Pipe, is a hit. She came up with the idea, developed it, and kept enthusiasm high. Because of her
success, you ask her to make a company wide presentation about how to maintain momentum and support during
the innovation process. Which of the following is Karen most likely to include in her presentation?

Correct

Good choice. You answered: Set goals that can be achieved quickly.

Retaining Employees:-

Question 1

Recent studies show that in every organization a certain number of employees are high risk; that is, theyre not
committed to their present employer and are not planning to stay for more than two years. What percentage of
employees is considered to be high risk?

Correct
Good choice. You answered:33 percent.

As many as 33 percent of employees one in three are "high risk."

REVIEW MATERIAL
Learn > Employee Retention Overview > The current state of employee commitment

Question 2

Recently, you promoted Joanne to a supervisory role. Her responsibilities have shifted from working with clients in
the field to managing business in the head office. She accepted the promotion primarily because of the salary
increase. In one of your monthly check-in meetings, she indicates that she misses working with clients more than
she thought. What is the best thing you can do to retain Joanne?

Correct

Good choice. You answered: Find a way to adjust her job duties to meet her interests.

Joanne became unhappy after her job duties switched from working in the field with clients to managing business in
the head office. If people want to leave because their work no longer interests them, supervisors can often find
ways to sculpt current roles to better match employees interests or to help people identify more satisfying
opportunities within the company.

REVIEW MATERIAL
Learn > Employee Retention Overview > Reversing the pattern

Question 3

Tanya is a human resources specialist with a management consulting firm. A local radio station has asked her to be
a guest on a phone-in show about employee turnover. To prepare for the show, Tanya decides to make a list of costs
associated with employee turnover. Which of the following is not a cost associated with employee turnover?

Correct

Good choice. You answered: When employees leave, remaining group members tend to lose interest in their jobs.

When an employee leaves, group members typically don't lose interest in their jobs, hence this is not a cost of
employee turnover.

REVIEW MATERIAL
Learn > Why Is Retention More Important Than Ever? > The cost of employee turnover

Question 4

You have been asked to make a presentation on the pressures that your company faces in trying to retain
employees. You have collected quotes from trade journals on this topic. You decide that one of the quotes is
inaccurate and will not be included in your presentation. Which of the following quotes is inaccurate?
Correct

Good choice. You answered: Company loyalty will always be a major factor in retaining staff.

It is inaccurate to say that employees will remain loyal to a company. Old assumptions about the employer as
protector and the employee as loyal devotees have evaporated. Many workers assume (correctly) that its up to
them to take charge of their own employability and careers even if it means moving from firm to firm to get the
best work and compensation package.

REVIEW MATERIAL
Learn > What Makes Retention So Challenging? > Upheavals in the world of work

Question 5

Kenneth is human resources manager of TraqDown Systems Inc., a developer of high-tech navigation equipment.
His staff is a mix of Generation Y workers and a smaller number of Boomer employees in key positions. Kenneth
develops a list of strategies to meet the needs of his employees and maximize retention. Which strategy is most
likely to appeal to the Boomers in his group?

Correct

Good choice. You answered: Introduce more flexible work opportunities, such as job sharing, sabbaticals, and
telecommuting.

Supporting job flexibility is a more effective strategy for retaining Boomers. Older workers are also interested in
unpaid time off and released time for community projects. Consider any of these offerings as well as "phased
retirement" for Boomers.

REVIEW MATERIAL
Learn > Retaining a Multigenerational Workforce > Strategies for retaining Boomers

Question 6

Doherty Savings is a midsized bank run primarily by men. When Rose became a senior vice president, she asked
Brenda to come up with strategies to retain women in the upper ranks. Rose liked all but one of Brendas
suggestions. Which suggestion did Rose consider as bad advice?

Correct

Good choice. You answered: Organize a seminar that teaches women how to succeed by using tactics that have
worked for their male counterparts in the organization.

This is not good advice. Although many older books on management advise women to act like men in order to
succeed, a wealth of more recent research contradicts this approach. Numerous female entrepreneurs offer more
flexibility, understanding, and an open management style all of which can give their corporations a vital
competitive edge.
REVIEW MATERIAL
Learn > The Special Challenges of a Diverse Work Force > Strategies for retaining women

Question 7

Which of the following approaches would not be an advisable strategy to address the diversity concerns of your
work force?

Correct

Good choice. You answered: Try to promote employees with diverse backgrounds but no experience.

Promoting unqualified employees is not an appropriate way to address diversity concerns. Such actions are
perceived as unfair and only serve to alienate everyone.

REVIEW MATERIAL
Learn > The Special Challenges of a Diverse Work Force > Strategies for retaining diverse workers

Question 8

You are a consultant at The Hiring Hand, a human resources and placement agency. One of your clients, an upper-
level manager at an office supply chain, asks you for some advice to help fill a senior position at her company. What
is the best tip you can give her?

Incorrect

Not the correct choice. You answered: Hire someone who shares your personality traits and outlook.

Many managers falsely assume that they will only be able to build a strong department or team by hiring people
who are like them. Diversity in personality, work styles, and decision-making approaches creates a richer corporate
culture and lets team members complement one anothers strengths.

The correct choice is: Hire someone who will feel fortunate to work at your company.

You should look for candidates who feel honored to receive a job offer from your firm people who will appreciate
the unique advantages that your organization has to offer. Theyll make the most loyal employees.

REVIEW MATERIAL
Learn > Hiring Right: The First Step to Retention > Avoid seeking the "hottest" prospects

Question 9

Jaffa is the manager of Homesite Care Corp. His nursing staff consists mainly of married women with school-age
children. He considers them to be a valuable resource and has implemented several strategies to retain their
services. One of these strategies is much less effective than the others. Which one is it?

Correct
Good choice. You answered: Offer higher salaries than similar organizations.

This strategy is the least effective option presented. While compensation is an important issue, you dont have to
outspend your competitors. Most people consider other things more crucial or attractive than big pay increases.

REVIEW MATERIAL
Learn > Stay Competitive > Compensation

Question 10

Carolyn is vice president at Soft Stuff Software. She has been asked for ideas on how to make the firm more fun for
the companys young, high-intensity staff. She considers several ideas. Which one is the best suggestion to present
to the company president?

Correct

Good choice. You answered: Put more emphasis on reshaping the department or team's culture.

Most companies have both a department or team culture (micro culture) and a company culture (macro culture).
However, it's the micro culture that plays the most powerful role in people's satisfaction with and enjoyment of their
jobs. The micro culture is where Carolyn has the best chance of understanding and influencing what it's like to work
at Soft Stuff Software.

REVIEW MATERIAL
Learn > Cultivate the Right Culture > The importance of microcultures

Question 11

You decide to develop a new training manual for supervisors on the basics of retaining valuable employees. You list
ideas and suggestions from your colleagues about how to enhance retention. Which of the following suggestions is
the best advice for improving retention?

Incorrect

Not the correct choice. You answered: Supervisors should not share their retention techniques.

Supervisors and managers should share retention techniques and experiences. By exchanging both success and
failure stories you will help each other avoid typical mistakes and you will be able to lend mutual moral support.

The correct choice is: Treat employees as people with full lives rather than as machines.
Coaching:-
Question 1
You are the regional sales manager for a stationery supply company. Which of the following
situations represents an appropriate coaching opportunity?
Correct
Good choice. You answered: Elise would like to move into a regional sales manager
position.

Coaching is an opportunity to contribute to another persons development in a two-way


partnership. You and Elise would work together to maximize her potential and help her reach
her goals.

REVIEW MATERIAL
Learn > Coaching Overview > What is coaching?
Question 2
When the human resources director observes Martas coaching practices, he notices that
she does not always use coaching appropriately. In which of the following situations is Marta
using coaching appropriately? When she uses coaching to
Correct
Good choice. You answered: Increase employee motivation and job satisfaction.

Through coaching, Marta and her reports will work together to lead to improved employee
performance. Over time, this partnership can motivate employees and increase their sense
of job satisfaction.
REVIEW MATERIAL
Learn > Coaching Overview > Why coach?
Question 3
You are the team leader of an information technology support team within a large insurance
company. Several members of your team have approached you with concerns about the
behavior of your coworker Ben. You think there may be a coaching opportunity. What should
you do first?
Correct
Good choice. You answered: Observe Bens behavior during team meetings and while on
service calls.

The first step is to understand Ben, his skills, and the situation. Observing an individuals
behavior in a neutral fashion, across different situations, will allow you to avoid premature
judgments and test your theories.
REVIEW MATERIAL
Learn > Preparation > Observe behavior
Question 4
As the supervisor at a fast-food restaurant chain you notice that Tonya, one of your most
experienced cashiers, is having trouble balancing her cash register at the end of each shift.
You and others have been staying past closing time to resolve the discrepancy. After
observing her performance for several days, you notice she seems to be making careless
mistakes, such as giving the wrong change. You wonder if she has become bored. What
would be the best way to share your observations with Tonya?
Correct
Good choice. You answered: Tonya, I observed you give the wrong change to several
customers this week and your cash register did not balance. Its hard on your coworkers
when they have to stay late while you balance your cash, and Id like to help.
Begin by describing the specific behavior you have observed with Tonya (e.g., giving
customers the wrong change and cash out of balance). Describe the behaviors impact on
group goals or other employees in a straightforward yet supportive way. Then give Tonya an
opportunity to respond before you draw any conclusions or offer any suggestions.
REVIEW MATERIAL
Learn > Initial Discussion > Link observed behaviors to impact on others
Question 5
Madeline is a project manager at a large software development firm. Madelines high-profile
project is significantly behind schedule. As her manager, you are using a coaching session to
explore options with the goal of getting the project back under control. What would be a
good question to ask Madeline to generate ideas for solving the problem?
Correct
Good choice. You answered: What do you think is the most critical issue on this project?

The use of an open-ended question invites Madelines participation and idea sharing. This
particular open-ended question will help you establish the priorities and allow Madeline to
elaborate on possible solutions.
REVIEW MATERIAL
Learn > Initial Discussion > Ask open-ended and closed questions
Question 6
As the office manager for a national medical services supplier, your direct report, Juan, is
ready to take on more responsibility. You want to help Juan achieve his career goals through
supportive coaching. Which of the following actions best represents supportive coaching of
Juan?
Correct
Good choice. You answered: Inviting Juan to help train a newly hired receptionist by acting
as a peer coach.

In supportive coaching, you act more as a facilitator or guide while encouraging employees
to find their own answers. By inviting Juan to become a peer coach, you are helping him
learn new skills through the experience of using his expertise to coach others.

REVIEW MATERIAL
Learn > Ongoing Dialogue > Customize your coaching approach
Question 7
Paul has been working directly with customers on the front line for several years. He is
looking for ways to increase his chances of becoming a supervisor and seeks your advice as
his manager. You say: "I think you should seriously consider this night school course on
employee relations. I took it myself and found it good for my career. How do you feel about
that?" Your response is an example of:
Correct
Good choice. You answered: Advocacy.

This is an example of advocacy because you offer your opinion and then you share a
relevant personal experience and encourage Paul to offer his own perspective.
REVIEW MATERIAL
Learn > Ongoing Dialogue > Blend inquiry and advocacy
Question 8
One of your account managers has just given a successful presentation to a prospective
client and youd like to provide feedback on the presentation. As her coach, what would be
the best way to voice your feedback to the account manager?
Correct
Good choice. You answered: The visuals you used in the presentation really impressed the
client.
Feedback should focus on describing a specific behavior and its impact on projects and/or
coworkers. In this example, you specifically comment on the effect the account managers
visuals had on the prospective client.
REVIEW MATERIAL
Learn > Ongoing Dialogue > Give feedback regularly and be specific
Question 9
Manuel is a middle manager at Salton & Friezen Inc., a food processing company. He notices
that Sharee, one of his direct reports, is having difficulty running meetings, and he decides
to help her. He draws up a coaching plan and spends time showing her how to run meetings
more effectively. What should Manuel do next? He should
Incorrect
Not the correct choice. You answered: Be careful not to change the coaching plan during
the life of the coaching process.

A good coach tries to continually improve the coaching process. Periodically ask what has
worked well in the coaching process itself and how the process could be improved. For
instance, do you and your coachee need to meet more or less frequently? Do you need to
use a more directive or more supportive coaching style than you originally assumed?

The correct choice is: Observe Sharees performance in a practical setting.

A coach should follow up coaching sessions with challenges designed to encourage the
coachee to demonstrate his or her new skills or knowledge. In this case, Manuel should
challenge Sharee to run a real or simulated meeting. If Sharee is still having trouble running
meetings, ask her what she needs from you to make more progress.
REVIEW MATERIAL
Learn > Follow Up > Be systematic about following up
Question 10
Eva is a key member of your team, but because she has so many skills, she tends to try to
do everything herself rather than delegate. As a result, shes overloaded and projects are
getting delayed. The biggest problem is that Eva doesnt even recognize there is a problem.
You consider whether Eva would benefit from immediate coaching. Whats the best thing to
do?
Correct
Good choice: Postpone coaching, but tell Eva that she needs to delegate more, giving her
the opportunity to change her behaviors independently.

If she fails to take corrective action, coaching may then be necessary.


Business Plan Development:-

Question 1
Jonathon, a product manager at Polymath Electronics, has been assigned to lead a team that
will develop a new product for the companys export market. Hes considering using a
business plan. What advice would you give him?
Correct
Good choice. You answered: Expect to discover more questions than answers while you
prepare the plan.

Preparing a business plan is part of a process that, at first, results in more questions than
answers. Jonathan will likely find that he must do research to answer those questions before
actually writing his plan.
REVIEW MATERIAL
Learn > The Business Plan Overview > The need for a plan
Question 2
Ursula is product manager at a major wireless communications company, has an idea for a
new service offering. She knows she must develop a business plan in order to gain the
support to implement her idea, but shes not sure how to begin. How should she begin
preparing her plan?
Correct
Good choice. You answered: Define the purpose and goals of the plan.

Ursula should begin by considering the overarching issues that shape the process of
preparing a business plan. She should start by defining her purpose for constructing the plan
and then determine what form the plan should take. Next, she should identify her business
plans audience and decide upon her resource needs.
REVIEW MATERIAL
Learn > Shaping the Process > Define your purpose
Question 3
Your friend, who works for an airline, is writing a business plan to develop a new route to
serve tourists from Southeast Asia. He wants to depart from the traditional business plan
model by combining two sections, the competitive analysis and the industry background,
and adding a section about the plans logistics. What would you advise him about the
changes he wants to make?
Correct
Good choice. You answered: Your changes are acceptable, just as long as the plan includes
descriptions of the opportunity, the context, the managers, and the financial risk and
reward.

A business plan may combine some elements, add new sections, and eliminate others, as
long as it covers all the critical information readers need to know for a particular business.
The plan should contain descriptions of: the business concept and why it will be successful;
the business and market context; the business managers and operations; and the financial
plan and its expected risks and rewards.
REVIEW MATERIAL
Learn > The Structure of the Business Plan > Structure overview
Question 4
You are preparing the executive summary of a business plan intended for venture capitalists
and other potential investors in your company. Which should you omit from the executive
summary?
Correct
Good choice. You answered: The financial assumptions.

You should omit the financial assumptions from the executive summary. The executive
summary is a concise presentation of the main points of the business plan; it is a kind of
abstract giving a brief overview of the business venture. Details such as the financial
assumptions are best covered in the financial plan and attachments sections of your plan.
REVIEW MATERIAL
Learn > The Executive Summary > Executive summary sections
Question 5
The management team of XYZ Engineering is writing a business plan to develop a new
version of one of the companys patented products. The plan will be reviewed by the chief
executive officer of XYZ's parent company. The team has written an executive summary that
focuses on a vision of the opportunity, rather than a summary of the entire plan. What would
you advise the XYZ management team about this approach to the executive summary?
Correct
Good choice. You answered: They can use this type of executive summary if they want to
capture the readers interest in the business opportunity.

While often the executive summarys purpose is to give readers a quick understanding of
the proposal, it can also serve to capture the readers interest in the business. This type of
executive summary functions more like a movie trailer than a condensed abstract in that it
encourages the reader to read on to see the whole picture.
REVIEW MATERIAL
Learn > The Executive Summary > Mission statement
Question 6
Sally is writing a business plan and asks your advice: I want to define the business
opportunity as I see it, in terms of the industry, the competitors, the market, and the
customers in that market. But I dont know what section of the business plan to put it in. In
which section would you tell Sally to place this information?
Incorrect
Not the correct choice. You answered: Business description.

The business description is a summary that focuses directly on the business concept itself
by giving a brief yet informative picture of the history, the basic nature, and the purpose of
the business.

The correct choice is: Business environment analysis.

The purpose of the business environment analysis is to show the reader what the business
opportunity is in the industry and market. The analysis should demonstrate that you have
identified an opportunity that solves a real customer problem. The key questions to be
considered in the business environment analysis are: What is the industry? Who are your
competitors within that industry? What is the market? and, Who are your customers within
that market?
REVIEW MATERIAL
Learn > The Business Environment Analysis > Understand the industry, competition, and market
Question 7
Chen thinks his company can profitably export industrial flooring to Western European
countries. In the Industry Background section of his business plan, hes covered the types of
products and services produced by the industry, the industry size and shape, and industry
trends. What important topic is missing from his industry background?
Correct
Good choice. You answered: Barriers to entry into the industry.

Barriers to entry into the industry should be addressed in the industry background section.
The section should describe the resources, knowledge, or skills needed to enter this industry.
It should also identify any restrictive federal or international regulations, large capital
requirements, or areas of sophisticated technical knowledge associated with providing the
products or services.
REVIEW MATERIAL
Learn > Industry Background > Key industry questions
Question 8
The following statements are excerpts from a business plan designed to attract investment
from a venture capital firm. Which of them does not belong in the competitive analysis?
Correct
Good choice. You answered: Growth rates in the industry have increased year over year
for the last four years.

The competitive analysis section identifies your competitors, differentiates your business
from similar offerings, and assesses competitive threats and advantages. Information on
industry trends belongs in the industry background.
REVIEW MATERIAL
Learn > Industry Background > The competitive analysis
Question 9
While reviewing a draft business plan submitted by the assistant manager of product
development, you notice a problem. The market analysis section in your colleagues
document contains the four parts listed below. Which one should he move from the market
analysis and place into the marketing plan?
Correct
Good choice. You answered: A road map for introducing the products into the market.

The market analysis section should focus on describing your target market: that group of
people who will choose to purchase and continue to purchase your product or service
because it solves an identified problem or meets a need. In contrast, a well-conceived
marketing plan functions as a road map for introducing your product or service to the target
market.
REVIEW MATERIAL
Learn > The Marketing Plan > Develop the marketing plan
Question 10
The operations section of a business plan should present to the reader the critical success
factors affecting how the company creates value. Which of these is considered the most
important factor?
Correct
Good choice. You answered: The breakeven point.

The most important success factor is the breakeven point the point at which unit sales
equal operating costs. The breakeven point determines how many units of the product or
service need to be sold to cover costs so the following units will produce a profit. Simply
stated, it identifies the point at which the business will begin to make money.
REVIEW MATERIAL
Learn > The Operations Plan > Breakeven point
Question 11
Your friend has been asked to write a management summary for a business plan, but he isnt
sure what a management summary is. In a panic, he calls you for help. You explain that a
management summary ...
Correct
Good choice. You answered: Consists of information on who will manage the
company/endeavor and how they will work together.

The management summary answers the readers questions about the management team
members and describes how they will work together to form an effective and successful
team.
REVIEW MATERIAL
Learn > The Management Summary > Describe the team
Question 12
A business plan contains this statement: Risden Associates intends to use the $250,000
raised for system development and programming (45 percent), marketing (35 percent), and
purchase of new operating facility (20 percent). In which section would you expect to find
this statement?
Incorrect
Not the correct choice. You answered: Operations plan.

This statement is not appropriate for the operations plan section. The operations plan
provides an overview of the daily activities of the business and the strategies that support
them.

The correct choice is: Financial plan.

Information regarding the amount of money needed and how it is intended to be used is
presented in the capital requirements section of the financial plan.
REVIEW MATERIAL
Learn > The Financial Plan > Capital requirements
Question 13
You are preparing a plan for the corporate investment committee of your company. They are
comparing business plans for several internal projects in order to decide which ones the
company should pursue in the coming year. When they look at the financial plan section of
your proposal, what will interest them most?
Correct
Good choice. You answered: The plans ability to achieve hurdle rate.

The investment committee members reviewing your proposal will want to know whether the
venture can achieve the companys hurdle rate (the minimum rate of return expected of all
projects).
REVIEW MATERIAL
Learn > The Financial Plan > Consider a reader's point of view
Question 14
Jackson asks Calum to get a copy of the pro forma financial statements to complete the
business plan. Calum e-mails you to say he is not sure what to look for. You respond that pro
forma financial statements are statements of ...
Correct
Good choice. You answered: Projected financial results.

The pro forma financial statements are projected statements what you believe will be the
future income
Budjeting:-

Question 1
Which of the following activities can proceed in the absence of a budget?
Incorrect
Not the correct choice. You answered: Rest Easy, a multinational hotel chain, intends to equip all its
guest rooms with high-speed Internet access within the next three years.

Since the Rest Easy Internet initiative will take place over the next few years, it would require a budget
with a long-term perspective. However, the Internet upgrade project could also involve drafting a
specific budget that focuses only on the resources necessary to complete the initiative.

The correct choice is: Worldwide Courier responds to a government plea for help in delivering food
and medical supplies to a city that has been devastated by a hurricane.

A budget does not apply to a rapid response to an unplanned and unforeseen incident. A budget is the
blueprint or action plan that translates strategic plans into measurable quantities that express the
expected resources required and anticipated returns over a designated period of time.
REVIEW MATERIAL
Learn > Budgeting Overview > What is budgeting?
Question 2
As the manager of global sales for Ellipse Pharmaceutical, you have already resolved to increase your
Asian sales by 10 percent in the coming year. You are currently reviewing the records of inquiries
generated by various forms of advertising. The information gained will be useful in determining your
future course of action. This strategy represents what step of the budget planning process?
Correct
Good choice. You answered: Evaluating and choosing options.

Once the goals have been determined, the next phase of the planning process is to evaluate options
for achieving the goal such as increasing advertising spending. Options available for attaining the
goal(s) are identified and predictions are made of the most likely outcomes for each option. Predicting
the costs and benefits of each option is part of this planning step.
REVIEW MATERIAL
Learn > Budgeting Overview > The budgeting process
Question 3
World Realty intends to increase its condominium sales worldwide over the next three years. As head
of finance, you have organized the budgets of each office into a master budget in order to monitor the
expansion process. In analyzing this budget, you recommend corrective action in one territory that has
an unfavorable variance in the (actual sales/expected sales) category. Which one?
Correct
Good choice. You answered: South America (90,000 / 110,000).

South America is the target of corrective action. Although it had the highest sales, it has an
unfavorable variance.
REVIEW MATERIAL
Learn > Variance in Budgeting > What causes variance?
Question 4
The annual report of Geomatic Telecommunications is being prepared. How will the companys gross
margin be calculated?
Correct
Good choice. You answered: Gross profit divided by sales.

Gross margin is correctly calculated as the gross profit divided by sales. Gross margin measures
profitability after direct production costs.
REVIEW MATERIAL
Learn > Budgeting Overview > Evaluating management's performance
Question 5
You are the head of finance at a global company that discovers and markets genetic-based therapies
for diseases. This sector is very competitive. Keeping one step ahead of your competitors means the
company must continually develop new products an expensive process. However, because of an
economic downturn you have also been charged with reducing costs over the next two years. Which
budget process is your best choice?
Correct
Good choice. You answered: Kaizen budgeting.

Kaizen budgeting is the most appropriate for this situation. The process, which derives from the
Japanese word kaizen ("continuous improvement"), attempts to introduce continuous improvement
into an organizations budgeting process.
REVIEW MATERIAL
Learn > Approaches to Budgeting > Kaizen budgeting
Question 6
As chief executive officer of Global Thermal Power Systems, Jacqueline is responsible for the approval
of the overall budget. Jacqueline has acknowledged the expertise of her various managers by having
them draft the budgets for their divisions. This process is an example of ...
Correct
Good choice. You answered: Participatory budgeting.

Jacqueline has used a participatory budgeting process. In this process, the managers responsible for
achieving the budget goals are included in setting their goals. However, the process is not iterative
since it does not begin with upper managements strategic goals and does not involve a negotiation
process.
REVIEW MATERIAL
Learn > Approaches to Budgeting > Traditional budgeting and alternate approaches
Question 7
It is December, and Nancy is in the process of drawing up the yearly budget for her company, the
Pack-n-Go Travel Agency. She is making a list of all the costs she expects to incur for the coming year.
Which of the following expenses would not go under the column labeled Fixed Costs?
Correct
Good choice. You answered: The sales representatives commissions.

Since the sales reps commissions will depend entirely on his success in selling vacation packages, his
commissions are a variable cost, not a fixed cost. A variable cost is one that alters in direct proportion
to changes in activity.
REVIEW MATERIAL
Learn > How to Categorize Expenses > Fixed and variable costs
Question 8
You are the comptroller for the Integrated Black Box Company with locations across the United States.
When you draw up your budget, which one of the following outlays would not be considered an
overhead cost?
Correct
Good choice. You answered: Wages for the assembly-line workers.

Wages for the assembly-line workers would not be considered an overhead cost. This expense is
directly tied to the products being produced and is not part of the overall company overhead.
REVIEW MATERIAL
Learn > How to Categorize Expenses > Allocated costs
Question 9
KarCorp Ltd. is a large automobile manufacturer with numerous divisions. The company proposes to
adopt activity-based budgeting (ABB) in order to obtain a clearer picture of costs and to plan more
effectively. In this situation, which of the following statements is false?
Correct
Good choice. You answered: ABB is appropriate for a single division.

ABB has to be adopted by and embedded into the entire organization; one division alone cant decide
to develop its own ABB system.
REVIEW MATERIAL
Learn > How to Categorize Expenses > Activity-based budgeting (ABB)
Question 10

Scenario Widgets Sold

Units Total Materials Revenue


Sold Cost ($) ($)

Current price 15,000 140,000 340,000

Increase price by 10
13,000 160,000 360,000
percent

Increase price by 15
11,000 180,000 400,000
percent
Which option will give you the best net revenue?
Correct
Good choice. You answered: Increase price by 15 percent.

This what-if scenario produces the best net revenue (revenue total materials cost). The net revenue
from this scenario is $400,000 $180,000, which equals $220,000.
REVIEW MATERIAL
Learn > Sensitivity Analysis > "What-if" scenarios
Question 11
Which of the following statements about the Balanced Scorecard is not true? The Balanced
Scorecard ...
Incorrect
Not the correct choice. You answered: Gives management a way to view the organizations progress
from the perspective of customer satisfaction.

The balanced scorecard does give managers a way to view the organization from the customer
perspective.

The correct choice is: Puts financial controls at the center of the planning process.

The Balanced Scorecard puts the strategic mission, rather than financial controls, at the center of the
planning process. The scorecard provides upper management a quick view of the critical factors
affecting the organization as well as information for translating the strategic vision into reality.
REVIEW MATERIAL
Learn > Linking the Budget to the Balanced Scorecard > The Balanced Scorecard and your budget
Question 12
The operating budget ...
Incorrect
Not the correct choice. You answered:Includes the cash flows.

The cash flows and balance sheet are aspects of the financial budgett.

The correct choice is: Provides the budgeted income.

The operating budget provides the budgeted income. This budget consists of the budgets from
functional areas such as R&D, design, production, marketing, distribution, and customer service.

Finance Essential:-
Question 1
Which one of the following statements about accrual accounting is incorrect?
Incorrect
Not the correct choice. You answered: Revenues are not deferred.
This statement is actually correct. In accrual accounting, both revenues and expenses are booked when they are
incurred, regardless of when they are actually received or paid. This system relies on the matching principle, which
helps companies understand the true causes and effects of business activities.
The correct choice is: Expenses can be deferred.
Expenses cannot be deferred in accrual accounting. Expenses are recognized in the same period as their associated
revenues. Income and expenses are booked when they are incurred, regardless of when they are actually received
or paid. Expenses can be deferred in cash-basis accounting, which is less conservative when it comes to expense
recognition.
REVIEW MATERIAL
Learn > Monitoring Financial Health > Accounting methods
Question 2
Skywire telecommunications equipment company has invested heavily in the past fiscal year to upgrade its
manufacturing equipment so that the company can take advantage of newer and more efficient technology. In
Skywires year-end income statement, the cost of the upgrade is presented as:
Correct
Good choice. You answered: Depreciation expense.
Depreciation is the appropriate expenditure category. Depreciation is used to estimate the consumption of an
asset over time, or to account for the diminishing value of equipment as time passes. A company would not
expense a piece of equipment in the first year of its purchase, but as the equipment is actually used over a set time
span.
REVIEW MATERIAL
Learn > The Income Statement > Decoding the income statement
Question 3
Which one of the following would be recorded as a liability on your company's balance sheet?
Correct
Good choice. You answered: Wages paid to the staff at the distribution plant.
Wages are a liability that are considered short term, or current. Short-term liabilities, which also include short-term
notes, income taxes, and accounts payable, typically have to be paid in a year or less.
REVIEW MATERIAL
Learn > The Balance Sheet > Decoding the balance sheet
Question 4
A new intern in your company is trying to learn something about cash flow. You decide to give him a quick overview
of how it works. Which of the following statements should you tell the intern?
Correct
Good choice. You answered: Cash flow projections may affect how you prepare the budget for the coming year.
Its a good idea to stay up-to-date with your companys cash flow projections, because they may come into play
when you prepare your budget for the upcoming year.
REVIEW MATERIAL
Learn > The Cash Flow Statement > Monitoring cash flow
Question 5
Geordie is head of operations for a bulk-food and consumer-goods retail company. When he returns to work from a
vacation, he wants to acquaint himself with the company's current financial position. Which financial statement
should Geordie look at?
Correct
Good choice. You answered: Balance sheet.
The balance sheet is appropriate, because it shows the company's financial position at a specific point in time. It
gives a snapshot of the company's financial situation - its assets, equity, and liabilities - on a given day. By doing so,
the balance sheet indicates how efficiently a company is utilizing its assets and how well it is mana ging its
liabilities in pursuit of profits.
REVIEW MATERIAL
Learn > Comparing the Three Financial Statements > The relationships among the statements
Question 6
James is North American sales director for a warehouse-style clothing and office-supply retailer. James relies on the
fast sales of his limited number of products to maintain his low prices and sustain a healthy cash flow. Which of the
following operating ratios is most useful to James?
Correct
Good choice. You answered: Days inventory.
Days inventory would be the most useful operating ratio for James. You calculate this ratio by dividing the average
amount of inventory on hand for the period by the cost of goods sold for the same period, then multiplying that
quotient by 365. Days inventory is a measure of how long it takes a company to sell the average amount of
inventory on hand during a given period of time. The longer it takes to sell the inventory, the greater the likelihood
that it will not be sold at full value - and the greater the sum of cash that gets tied up.
REVIEW MATERIAL
Learn > Measuring Financial Health > Operating ratios
Question 7
A retreat of departmental managers to sort out con flicts that arise from competition over limited resources could
occur as part of ...
Correct
Good choice. You answered: Bottom-up budgeting.
Bottom-up budgeting, whereby managers put together budgets that they feel will best meet the needs and goals of
their respective departments, often requires department heads to work closely with colleagues that may be
competing against them for limited resources. It's best to be both cooperative and assertive about your unit's needs
during this process.
REVIEW MATERIAL
Learn > The Budget Process > Bottom-up budgeting
Question 8
Which of the following scenarios illustrates a best practice for preparing a budget?
Incorrect
Not the correct choice. You answered: Ira plans to boost sales by reducing the retail price to its lowest level ever.
Because the new price is substantially lower than last year's, Ira decides not to base his budgetary assumptions and
scenario's on last year's sales results.
If Ira plans to boost sales by changing prices, he should start with last year's sales figures and then increase them
by the appropriate amount.
The correct choice is: Bonnie has a file entitled "Budget Diary," where she stores paperwork pertaining to her
budget deliberations and assumptions.
Bonnie is wise to document her assumptions. It's easy to lose track of your assumptions as you're entering numbers
into the required budget format. By documenting your assumptions, you stand a better chance of being able to
explain them and change them easily.
REVIEW MATERIAL
Learn > Articulating Your Assumptions > Defending your assumptions
Question 9
The facility manager of Grand Auto Manufacturing is considering re-equipping the New Jersey assembly plant. The
$90 million cost will produce an annual savings of $15 million over the eight-year expected lifetime of the
machines. What is the payback period?
Correct
Good choice. You answered: Six years.
The payback period, calculated as the total investment cost divided by the annual savings, is 90 divided by 15,
which equals 6.
REVIEW MATERIAL
Learn > Return on Investment and Payback Period > How long will it take for an investment to pay off?
Question 10
When you evaluate a potential investment you need to get a more realistic picture of the anticipated benefits.
Which of the following would you use to calculate the time value of money?
Correct
Good choice. You answered: Net present value.
Net present value is used to calculate the time value of money by providing a method for expressing future dollars
earned in terms of current dollars. A dollar you receive today is worth more than a dollar you receive five years from
now. That's because the dollar you receive today can be invested elsewhere, which means you'll have more than a
dollar by the fifth year.
REVIEW MATERIAL
Learn > Net Present Value and Internal Rate of Return > The time value of money
Question 11
As chief executive officer of Giant Chemical Corp., Manfred is contemplating acquiring a company that
manufactures artificial he art valves and hip joints. To reach his decision, Manfred undertakes a breakeven analysis.
Which of the following determinations represents the contribution portion of his analysis?
Correct
Good choice. You answered: Money that can be used to pay the fixed costs.
The contribution portion of the breakeven analysis - the unit revenue minus the variable costs - is the sum of money
available to contribute to paying fixed costs, such as those entailed by an investment in the new company.
REVIEW MATERIAL
Learn > Breakeven Analysis > The breakeven calculation
Question 12
As a financial officer with an aircraft manufacturer, Rob begins tracking the financial performance of a new model of
business jet. He notes an increase in equipment failure on the new assembly line in March. When the situation has
not improved by September, Rob redirects funds available for other assembly lines toward the new jets assembly
line. When Rob issues the next quarterly budget in December showing a decrease in expected performance of the
new assembly line, senior managers express concerns. What is the likely reason behind senior managements
reaction? Rob failed to ...
Incorrect
Not the correct choice. You answered: Regularly reassess the budget forecast.
Rob did reassess the budget forecast in his September update. Updating forecasts regularly ensures that managers
always have the latest and most accurate information on which to base decisions about whether to rethink
initiatives.
The correct choice is: Communicate with upper management.
Rob did not communicate with upper management. Thus, senior managers could not make appropriate adjustments
in the overall company budget or provide Rob with direction on whether or how to address the shortfall.

Leading and Motivating:-

Sean is a middle manager, known for running a tight ship in a government health agency. He
plans meticulously, completes projects on time and on budget, and equitably distributes
workloads. His department is facing a merger and Sean must compete for his own job. He
wants to assure his employer that he would be a competent leader in a time of change. In
his interview, he should
Incorrect
Not the correct choice. You answered: Summarize the ways in which he has resolved
routine production problems quickly, without having to immerse staff in lengthy
consultations or involve upper management.

Controlling staff and problem solving are important managerial skills, but Sean needs to
highlight his leadership skills in times of change. Although Seans problem-solving ability is
an important managerial skill, his style is not consultative. An autocratic style of decision
making for routine tasks is not going to engage and motivate staff during a time of change.

The correct choice is: Describe a strategy that he believes will align staff in the merged
department and promote effective teamwork.

The key to good leadership is the ability to set a direction, align staff, and motivate them
during times of change. Managing and leading are distinct but complementary tasks.
Managing involves coping with complexity; leadership, coping with change.
REVIEW MATERIAL
Learn > What Leaders Really Do > Managing versus leading
Question 2
Italia Foods has been an importer of Mediterranean gourmet fare for 72 years. Facing
increasing competition, the family firm seeks a new chief executive officer someone who
will introduce a more contemporary style of leadership. Which candidate best exemplifies
outstanding leadership qualities? The candidate who
Correct
Good choice. You answered: Knows their weaknesses and can remain focused on their
goals despite obstacles.

What distinguishes outstanding leaders is their degree of emotional intelligence, or the


ability to manage oneself and ones relationships effectively. The candidate who recognizes
their weaknesses demonstrates a high degree of emotional intelligence in the competency
of self-awareness, while the ability to remain persistent and focused on goals under difficult
circumstances exemplifies strength in the emotional intelligence capability of motivation.
REVIEW MATERIAL
Learn > Skills and Characteristics of Leaders > Emotional intelligence capabilities
Question 3
Sara personifies the characteristics of an effective leader. She is
Incorrect
Not the correct choice. You answered: Quick to judge and unconcerned with company
politics.

An effective leader is level-headed in the midst of turmoil and confusion and is also
politically astute.

The correct choice is: Caring and at ease with ambiguity.


An effective leader has the demonstrated ability to empathize with others needs and
concerns. She is also willing to take calculated risks and change her mind when new
information comes to light.
REVIEW MATERIAL
Learn > Skills and Characteristics of Leaders > Mapping emotional intelligence to leadership traits
Question 4
Ahmed manages a group of highly trained, experienced, and self-motivated engineers at
Munchen Electronics. Business has been slow in recent months. This fact has begun to
destroy team spirit and left many members of his team feeling depressed about their jobs.
What is the best leadership style Ahmed can use in this situation?
Correct
Good choice. You answered: Affiliative.

The affiliative, or people first style, is best used when team coherence is important or in
times of low employee morale. While this style may sometimes permit poor performance
among employees to continue unchecked, the engineers in Ahmeds team are highly self-
motivated and thus their performance may not be a significant issue.
REVIEW MATERIAL
Learn > Adapting Your Leadership Style > Leadership styles
Question 5
Eleanor is a manager at Motivaide Inc., the countrys sixth-largest manufacturer of walkers
and canes. She is writing a vision statement for all employees in the company. Which of the
following statements would be most effective?
Correct
Good choice. You answered: We will become the largest manufacturer of walkers and
canes in the country within a decade.

An effective vision must be compelling yet realistic. The most successful visions illustrate
a future that is so much better than the current state that people will gladly undertake the
effort and sacrifices necessary to attain it.
REVIEW MATERIAL
Learn > Crafting a Vision That Others Will Follow > Characteristics of an effective vision
Question 6
After 28 years as a wholesale supplier to restaurants, Banff Foods is moving into the retail
market. The vice president, John Peck, wants to get everyone behind the companys new
direction. What is the best way for him to communicate his vision? He should
Incorrect
Not the correct choice. You answered: Tell suppliers how the new strategy will improve
profitability.

Talking about the impact on company profits is not the best way to communicate Chads
vision to these stakeholders. Instead, Chad should clearly spell out the benefits of his
alternative future in terms that are most relevant to his impact on suppliers, like decreased
costs.

The correct choice is: Deliver the same message repeatedly in different formats.

Repetition is the best approach to communicate a vision since it will help a leader spread
and reinforce a consistent message. On a rational level, repetition helps everyone stay
focused and understand exactly what they are supposed to do to help realize the vision.

REVIEW MATERIAL
Learn > Aligning People Through Successful Communication > Communicate your vision
Question 7
Jane is a conscientious employee but she appears unenthusiastic about her companys new
vision for the future. What is the best approach her manager can take to get her motivated?
Her manager should
Correct
Good choice. Your answered: Ask Jane what motivates her best and plan accordingly.

It is best to find out what motivates Jane and plan accordingly. All people have individual
drives, needs, and desires. Because of this, they will be motivated by different factors.
REVIEW MATERIAL
Learn > Motivating Others > Tap into internal sources of motivation
Question 8
You have been asked to speak at a local service club luncheon on the topic of motivation in
the workplace. Which of the following would most likely be part of your presentation?
Correct
Good choice. You answered: A sense of achievement on the job, feedback on the quality of
work, and a feeling of being responsible are examples of internal motivational factors.

All of these are topics would likely be included because they are examples of internal
motivational factors. The work itself and providing opportunities for growth and learning are
other examples.
REVIEW MATERIAL
Learn > Motivating Others > Tap into internal sources of motivation
Question 9
Lucinda has shown talent in the past, but since the company rolled out its new vision
statement, she has been uncooperative and difficult. Her supervisor, April, has noticed the
problem. What is the best course of action for April? She should
Correct
Good choice. You answered: Ask Lucinda how she thinks the problem might be solved.

April should work with Lucinda to help her solve the problem. She should begin by inviting
Lucinda to suggest tactics for resolving the issue. The resolution they mutually determine
should appeal to Lucindas drives.
REVIEW MATERIAL
Learn > Energizing Difficult People > A three-step approach to motivating a problem employee
Question 10
Avid & Bryant Construction is finishing a major contract on the New York waterfront. There
have been many delays with work permits and cost overruns. The working environment has
become fractious; staff are pointing fingers and assigning blame. As project leader, youre
accountable and under increasing pressure to get the project back on track. In order to
create a holding environment, you would
Correct
Good choice. You answered: Create an organizational space for team members to discuss
challenges and clarify assumptions about timelines and expectations.

To help motivate people without immobilizing them, effective leaders create holding
environments: safe organizational spaces in which the conflicts, emotions, and stresses
associated with a change can be worked out. In the earlier stages of implementing a vision,
a leader may establish a holding environment to talk openly about the initial challenges, to
frame and debate issues, and to clarify assumptions.
Presentation skills:-
Question 1
The programming code of a companys recently launched Internet browser is unique to the
industry. You wish to explain in detail how the programming code works. What is the best
way to explain these technical details?
Incorrect
Not the correct choice. You answered: Make a persuasion presentation.
A persuasion presentation is used to provide reasons to pursue a particular idea or path and
is not advisable for presenting detailed information.
The correct choice is: Create a written communication.
Use written forms of communication to capture detailed information, such as a unique
computer code. If a presentation is used to communicate technical information, audiences
will not remember the important details.
REVIEW MATERIAL
Learn > Presentation Overview > Different types of presentations
Question 2
A presentation that begins with some in-house jokes and anecdotes as a lead-in to a
description of the latest research findings would be best suited to which scenario?
Correct
Good choice. You answered: A presentation of research results during the weekly company
meeting of technical staff and salespeople.
An informal, lighthearted presentation is appropriate for a weekly meeting of colleagues.
REVIEW MATERIAL
Learn > Laying the Foundation for Your Presentation > Understand your presentation's context
Question 3
Rita is a plant manager at a water treatment company. The local town council has asked for
a general update to the community on the companys progress in cleaning up the
waterways. Rita prepares a high-level status report and also includes several pages of highly
specific scientific measurements. What mistake did she make? She ...
Correct
Good choice. You answered: Failed to understand the audience.
By presenting detailed technical information, Rita failed to understand that her audience
may not have the expertise required to comprehend her report. Since the council was
expecting a general progress report, Rita provided more information than the audience
needed to hear.
REVIEW MATERIAL
Learn > Laying the Foundation for Your Presentation > Know your audience
Question 4
As the head of Healthy Day Medical Care, you are writing a talk on nursing involvement in
prenatal care. You will be delivering this lecture at a meeting of pediatricians. You are having
trouble getting started. What should your first step be?
Correct
Good choice. You answered: Identify the central message.
The first step in preparing any presentation is defining the key message: what you want
people to remember and what action (if any) you want them to take. The key message flows
directly from your objective. While your presentation may contain several supporting
arguments and ideas, it contains only one key message.
REVIEW MATERIAL
Learn > Deciding What to Say > Stages of preparation
Question 5
Bruce is president of Upper Minas Insurance. He is speaking to a convention of several
hundred insurance agents being held in his home city. Bruce begins his talk with, Good
morning. Take a look around the room. Of the 200 or so who are here, chances are, within
the next four years, six of you will fake an injury and seek long-term workmans
compensation. And four of you will get away with it. This opening is an example of a ...
Correct
Good choice. You answered: "Hook."
A "hook" is a question, comment, relevant story, statement, or example that engages the
audience during the opening minutes of a presentation. By opening his presentation using a
hook, Bruce has a good opportunity to catch the attention of his audience.
REVIEW MATERIAL
Learn > Organizing Your Presentation > Start strong
Question 6
A need or problem statement ...
Correct
Good choice. You answered: Develops a scenario that can be solved.
The need or problem statement is the second part of your presentation. It can be used to
develop a clear need or problem that you and the audience will solve together.
REVIEW MATERIAL
Learn > Organizing Your Presentation > Problem statement
Question 7
Shannon is organizing a presentation on urban parklands. She plans to begin with startling
statistics that relate the lack of urban parkland to an increased rate of depression. Next,
shell make it clear why the audience should care about establishing green belts in cities.
Shannon will then list actions taken in other cities that faced a similar situation. What does
her presentation lack?
Incorrect
Not the correct choice. You answered: The solution.
SBy listing actions taken in other cities that faced a similar situation, Shannon has provided
her audience with appropriate solutions.
The correct choice is: The call to action.
Shannon's presentation does not include the action step - the call to action. A call to action
is a wrap-up of the presentation. Positioned after a solution has been presented, a call to
action reiterates the key message, integrates the opening points into the closing comments,
recommends action, suggests agreement, obtains an audience commitment, and provides
closure to the issue.
REVIEW MATERIAL
Learn > Organizing Your Presentation > Call to action
Question 8
John has learned that the president of his company requires more presentation time at the
annual meeting. As a result, John has to cut his own presentation from 45 minutes to 25
minutes. He considers a number of options. Which is the best response to deal with his
limited time frame?
Correct
Good choice. You answered: Reduce the number of points in the talk.
A presentation should only be long enough to be clear and complete within the allotted time.
If there is not enough time available to make a point clear or acceptable to your audience,
save the point for another presentation. It is better for John to make fewer points and make
them well. For added comfort, he could plan beforehand what to delete if time is cut short.
REVIEW MATERIAL
Learn > Organizing Your Presentation > Duration
Question 9
You have a major presentation coming up. To improve the effectiveness of your presentation,
you practice in front of your colleagues and ask them for feedback. Which is the best
example of good feedback?
Correct
Good choice. You answered: "Try wandering into the group more. It helps get us involved."
Advising someone to wander into the group is good feedback. Avoid standing still behind a
podium. Walking around your audience or using movement such as relaxed gestures
provides control and more involvement.
REVIEW MATERIAL
Learn > Presenting Effectively > Manage your delivery
Question 10
Marion has been given the task of speaking to the senior managers about a proposal to
consolidate the companys production facilities in Boston. The idea is being received with
hostility by some managers whose facilities would be closed. How could Marion best handle
questions in her presentation?
Correct
Good choice. You answered: Take questions at the end of the presentation.
Taking questions at the end of the presentation should allow Marion to complete her talk on
schedule and make sure that the audience has the whole picture.
REVIEW MATERIAL
Learn > Handling Questions > When should you answer questions?
Question 11
Bob and Gloria are involved in planning the logistics of a group presentation on the details of
an early retirement package for a large consumer foods company. Their presentation plan is
summarized below. What part of their plan would you advise against?
Correct
Good choice. You answered: They allow an opportunity for every speaker to respond to
every question from the audience.
Allowing every speaker to comment on every question is not an advisable strategy for a
group presentation. Individual speakers should be prepared to answer only those questions
that relate to the topic they addressed in their presentation. However, they could also follow
up if another speaker needs help answering a question.
REVIEW MATERIAL
Learn > Making Group Presentations > Assign presenters
Question 12
Raoul is a risk management specialist with an international telecommunications company.
He employs the strategies listed below to help evaluate his highly technical, industry-specific
presentation. All of these strategies are good except one. Which one is it?
Incorrect
Not the correct choice. You answered: He had someone in the audience gauge reactions
to the presentation.
Having another person in the audience take notes on audience reaction is a useful
evaluation tool.
The correct choice is: He gave trial presentations to people in other industries.
Raoul's topic is industry-specific and technical. It is advisable he practice his presentation on
a test audience with a profile similar to that of his intended audience. If Raoul is presenting
to experts, his test audience should contain experts.
Change management:-
Question 1
Last year, the Pitz Winery increased production to include some new grape varieties. As part of their expansion,
they found that the old "top-down" style of management was no longer appropriate and have moved to a
management style that lets employees get more involved. What kind of change is this?
Correct
Good choice. You answered: Cultural change.
Cultural change focuses on a company's operating values, norms, behaviors, and the relationship between its
management and employees.
REVIEW MATERIAL
Learn > The Dimensions of Change > Types of change
Question 2
Constantine is a unit manager who has successfully led his team through several change initiatives over the past
ten years. In doing so he has shown some of the important characteristics of an effective change manager. Which of
the following characteristics would he not have displayed?
Correct
Good choice. You answered: Acceptance of barriers that may interfere with the change.
A successful change manager does not accept barriers but rather is able to eliminate problems and barriers that
interfere with the change initiative.

REVIEW MATERIAL
Learn > The Dimensions of Change > Characteristics of effective managers of change
Question 3
You are having lunch with a group of old college friends, all of whom are in business management. You mention that
your company is planning a major reorganization and you want to make sure that the people in your unit are
prepared for the upcoming changes. Who gives you bad advice?
Correct
Good choice. You answered: Joaquin says you should avoid confusion by providing information on a need-to-
know basis.

Good change managers share information freely. Information is the lifeblood of any organization. During times of
change, getting and disseminating information is critical to operating effectively, flexibly, and quickly.
REVIEW MATERIAL
Learn > Being Ready for Change > Prepare your group for change
Question 4
The detective agency of Hyerd, Gunn, and Associates is reorganizing the reporting policies for its operatives in
order to meet new regulations. There will be some resistance from the field because the changes will involve more
paperwork. Which of the following steps is the first thing the partners should do to enlist the operatives' support?
Correct
Good choice. You answered: Clearly define the problem prompting the change.

The starting point of any effective change effort is a clear definition of the business problem. Problem identification
answers the most important question employees will ask: Why must we undergo change? The answer lays the
foundation for motivating all employees in an organization.
REVIEW MATERIAL
Learn > Change Management > Mobilize energy
Question 5
You write a newspaper column that answers questions on business management. One of your readers asks you
where his company's managers should focus their attention during an upcoming, year-long change initiative. What
is your reply?
Correct
Good choice. You answered: Short-term, results-driven programs.

Short-term, results-driven programs generate wins such as providing evidence that sacrifices are worth it,
rewarding people involved in the change effort, and helping to fine-tune vision and strategies. They also build
momentum that turns neutrals into supporters.
REVIEW MATERIAL
Learn > Change Management > Create near-term wins
Question 6
Bernardo is a business analyst for a large industrial firm that is going to re-tool its assembly line. The plant manager
is working on the implementation plan and has asked Bernardo for some helpful hints. Which piece of advice would
Bernardo not give to the plant manager?
Correct
Good choice. You answered: Obtain all of the content for the implementation plan from upper management.

It is important that people at all affected levels help to build the plan. It should not be imposed upon them. If they
have been allowed to participate in its development, then they will be more likely to support it.
REVIEW MATERIAL
Learn > Implementing Change > Enlist the support and involvement of key people
Question 7
The Shorte Power & Light Company has just launched a new program called "Formulate Useful Solutions for Energy
Savings" (F.U.S.E.S.). It is aimed at streamlining operations and motivating consumers to find innovative ways of
reducing power consumption. What type of communication should not be part of the Shorte F.U.S.E.S. strategy
when dealing with this change initiative?
Correct
Good choice. You answered: One-way communication.

The company should make communication an ongoing, two-way proposition. While it is important to share
information, it is equally important to listen to what others have to say.
REVIEW MATERIAL
Learn > Communicating Change > Delivering the message
Question 8
Yvette is the new manager at an animal shelter. She launches a project to develop a more effective Web site for the
shelter but encounters resistance from Juan and Lynda, who helped to develop the original site. Which of the
following is Yvette's best course of action in this situation? She should ...
Incorrect
Not the correct choice. You answered: Use humor to diffuse the tension when Juan and Lynda tell her their
concerns.

Managers should listen carefully to the concerns of resisters, explore their fears, and take their comments seriously.

The correct choice is: Evaluate her own leadership style.


As you consider resisters, be sure to evaluate what part you may be playing in causing their resistance. Its possible
that your approach to managing change or your leadership style may be threatening to others, thereby causing
unnecessary friction and conflict.
REVIEW MATERIAL
Learn > Understanding Reactions to Change > Address resistance
Question 9
Elena is leading a discussion group intended to help the participants understand reactions to change. She has asked
them for some of the reasons people resist change. Which of the following responses is the least accurate?
Incorrect
Not the correct choice. You answered: Resisters believe that change is unnecessary.

This would be an accurate response. People can sometimes feel that the changes being made are inappropriate.

The correct choice is: Resisters fundamentally want to undermine change efforts.

While some resisters can undermine a change effort, it is shortsighted to think all resisters will, or even
fundamentally desire to, do so. Resistance implies energy energy that can be worked with, and possibly
redirected. Instead of viewing resistance as an obstacle, try to understand its sources and motives.
REVIEW MATERIAL
Learn > Understanding Reactions to Change > Rethink resisters
Question 10
You are feeling very stressed by a long series of changes taking place within your company. You decide to get some
advice from your next-door neighbor, who happens to be a psychologist. Which of the following activities would she
be unlikely to recommend?
Incorrect
Not the correct choice. You answered: Assess the changes objectively to determine their validity and worth.

This would be valid advice because it would help you to see the changes impartially.

The correct choice is: Place more emphasis on your work and determine how you can be more effective.

You would not want to invest all your emotional energy into addressing workplace concerns. It can take months for
issues to be resolved so it is important to strive for equilibrium between your work and personal life.

Developing employees:-

Question 1
After six months as a manager at Global Pharmaceutical, Bruno consults with
his mentor to seek advice on strategies for developing employees. What would
be the best advice to give Bruno?

Correct
Good choice. You answered: Consider emphasizing informal, on-the-job
learning opportunities over formal classroom training sessions.

Informal, on-the-job opportunities (such as "stretch" assignments, coaching, job


rotations, etc.) are an effective way to develop employees. Many managers now
emphasize this type of learning in their development plans, instead of focusing
primarily on formal training.

REVIEW MATERIAL
Learn > Why Develop Employees? > A new approach
Question 2

A retired friend is reminiscing about his days as a corporate executive. He


mentions several ways in which employee development has changed over the
years. Which of the following would he give as an example of the new approach
to employee development?

Correct
Good choice. You answered: Make development the responsibility of all
supervisors, not just Human Resources.

The new approach to employee development recognizes that employee


development is every manager's responsibility.

REVIEW MATERIAL
Learn > Why Develop Employees? > A new approach
Question 3

It is Take Your Child to Work Day, and your daughter is spending the day at the
office with you and your staff. She asks you how your company goes about
helping employees to improve their skills. What would you be unlikely to tell
her?

Incorrect
Not the correct choice. You answered: "We ask employees to take charge of
the process of managing their careers."
Employees themselves do own the process of effectively managing their
careers.

The correct choice is: "We reserve career development discussions for times
of organizational change."

It is important to discuss development needs at frequent intervals, not just


when major changes are taking place.

REVIEW MATERIAL
Learn > Addressing Developmental Needs > Your role in development
Question 4

You are having coffee with friends in Human Resources and the conversation
centers around strategies for differentiating between employees. Everyone has
a different opinion about the best way to accomplish this. Some of the
controversial methods generate discussion. But a newcomer to the team
suggests a method that is quickly rejected as inappropriate practice by the
others in the group. Which one is it? Differentiating based ...

Incorrect
Not the correct choice. You answered: Primarily on performance.

Many companies use a system of differentiation based primarily on historical


performance. A more complete performance picture could be formed by also
considering employee potential.

The correct choice is: On managers' judgments of "best" and "worst."

Differentiation should not be a process by which you pass judgment on which of


your employees is the "best" or the "worst."

REVIEW MATERIAL
Learn > Acknowledge Employee Differences > Differentiating may be difficult
Question 5

You are proofreading a checklist of the performance criteria your company will
use to evaluate employees. You notice that one of the items is actually used to
gauge employee potential, not employee performance. Which item actually
describes employee potential?

Incorrect
Not the correct choice. You answered: Acts on feedback.

Acting on feedback is a measure of employee performance.

The correct choice is: Demonstrates leadership ability.

The demonstration of leadership ability is a measure of an employee's potential


to fill future positions of greater responsibility.

REVIEW MATERIAL
Learn > The Performance and Potential Grid > Evaluate employee potential
Question 6

You are presenting a paper at a conference on employee motivation. You're


reviewing how companies use intrinsic and extrinsic motivational factors to
encourage employees to reach their full potential. What would you give as an
example of an extrinsic factor?

Correct
Good choice. You answered: Job status.

Job status is related to the job's environment and as such is an extrinsic


motivation factor.

REVIEW MATERIAL
Learn > Keeping Top Employees Motivated > How do you motivate employees?
Question 7

Your new department has a large group of employees who meet expectations
but who are not outstanding performers. You are a little disappointed until a
colleague points out that these are the kind of people who form the backbone
of the organization. Which of the following would not describe those
employees? They often ...

Correct
Good choice. You answered: Are eager to advance in the organization.

Often, reliable performers are not eager, or even able, to advance in the
organization. Such employees should be allowed opportunities if they are
interested but should not be pushed.

REVIEW MATERIAL
Learn > Growing Competent Employees > Develop solid contributors
Question 8

You are attending a management seminar on employee development. The


instructor is outlining the use of the Return on Management (ROM) ratio. Which
of the following statements would he be unlikely to make about the ROM ratio?
ROM ...

Correct
Good choice. You answered: Measures unproductive energy released against
time and attention invested.

The Return on Management ratio measures productive (not unproductive)


energy released against management time and attention invested. It can help
you decide whether you are investing energy in activities that best contribute
to your organization's productivity and overall performance.

REVIEW MATERIAL
Learn > Acting on Underperformers > The return on management ratio
Question 9

Production at the local telephone assembly plant has dropped significantly over
the last year. You attribute the problem to a number of underperformers in the
operation. You know that the first step in dealing with them is attempting to
move them up to an acceptable performance level. What would be the best
way to go about it?

Correct
Good choice. You answered: Provide candid feedback to the underperformers.

One of the ways to help underperformers achieve an acceptable performance


level is to provide coaching and candid feedback.

REVIEW MATERIAL
Learn > Acting on Underperformers > The second step: move them upor out
Question 10

You wish to define development plans for your direct reports. You have carried
out performance reviews in the past but this is the first time you have
conducted a career development discussion. You ask for advice from your
coworkers. Which colleague gave poor advice?

Correct
Good choice. You answered: Lisa said it is important to complete a
development plan in one meeting.

It is poor advice to try to complete a development plan in one meeting. A


career development discussion is a collaborative process. Therefore, it will likely
take more than one meeting to perfect a development plan.
Delegating:-

Question 1

You have recently been promoted to the position of manager of a busy payroll department. Which of the following is
the best argument for doing the work yourself rather than delegating to your staff?

Correct

Good choice. You answered: None of the above.

Question 2

As manager of the accounting team at FirstCare Medical Supply, you want to begin delegating more assignments to
your staff. Which of the following approaches will help you to ensure effective delegation?

Correct

Good choice. You answered: Allow employees the opportunity to work on delegated assignments that are high
profile within FirstCare.

Question 3

You manage the Internet services team for a financial planning firm. Your team is redesigning the corporate Web
site and you want to create excitement about the redesign within the organization. You ask Leon, one of your senior
team members, to organize a promotional campaign about the redesign. You put him in control of all aspects of the
campaign, including creating the announcement, writing an article for the newsletter, and planning the site launch.
This is delegation by ...

The correct choice is: Project.

Question 4

You have been hired as sales manager for a high-tech development firm, largely because of your vision and
knowledge of the industry. Once you have settled into your new position, you decide to delegate more responsibility
to team members who have expressed a desire to be promoted. Which of the following assignments should you
delegate?

Correct

Good choice. You answered: Organizing and taking full responsibility for this years annual sales conference.

Question 5

You are the manager of a team of production line supervisors at an automobile manufacturing company. You receive
a request to review and provide input on several new line improvement proposals. Since you have little time on
your hands, you decide to delegate the task to one of your four direct reports. Which team member would be the
best choice?

Correct

Good choice. You answered: Eli, because he has performed well on a task you delegated in the past.

Question 6

You are a restaurant manager for a fast-food chain. Marta, one of your most experienced full-time cashiers, has
become bored with her job. You would like to give her more responsibility by delegating the task of scheduling the
part-time cashiers. When you set up this assignment with Marta, what is not a key part of the delegation
discussion?

The correct choice is: An explicit understanding of career implications and salary expectations.

Question 7

You are the corporate accounts manager for a large accounting firm. You have been developing a new high-stakes
customer loyalty project if it fails, the firm will lose a great deal of money and market share. You are planning to
delegate the project to Justin, a senior account manager with excellent decision-making skills and a proven track
record of success. What level of authority would you grant Justin? Ask Justin to ...

The correct choice is: Research the issue, prepare a report, create alternatives, and decide on a course of action.
You approve the proposal before it is implemented.
Question 8

Two months after delegating an assignment to Lila, you follow up to check her progress. When you reflect upon how
you have monitored Lila, you conclude that you were sometimes more intrusive than supportive. What are you
doing that could be seen as intrusive? You ...

Correct

Good choice. You answered: Volunteer advice on the best methods of accomplishing her most complex tasks.

Question 9

As director of product development at Liberty Trust Bank, you delegate your credit card enhancement project to
Zoltan. Several weeks into the project, Zoltan's project plan gets derailed because other business units are not
producing the required deliverables on time. He sends you an e-mail asking you to intervene with the other
business units on his behalf so he can get the deliverables he needs. What would be the best way to respond?

Correct

Good choice. You answered: Compliment Zoltan on his work so far and coach him on project management skills.

Question 10

You set up a meeting to evaluate Anne, a direct report who has just completed a key delegation assignment:
designing the new hire program. You begin by asking her opinions of the experience. Following this, you describe
what you liked about her program design and how it met your expectations. Next, you discuss the problems that
occurred to try to determine who was responsible for each of them. Finally, you help Anne create an action plan for
her further growth and development. What part of your discussion was inappropriate?

Correct

Good choice. You answered: You assessed the problems to determine her responsibility.
Global Collaboration:-

Question 1

You are running a seminar on global collaboration and have asked students to list some possible benefits. Which
student gave an incorrect answer?

Correct

Good choice. You answered: Sandra said that global collaboration makes it easier to build trusting relationships.

Question 2

Frederick has been growing increasingly frustrated during a visit to an overseas partner. He has made a number of
cultural gaffes and is having a hard time adapting to the local business style. A colleague traveling with him
suggests assessing national cultural differences using Geert Hofstedes framework. Which of the following is not
one of the five dimensions of this model?

The correct choice is: Low numeracy to high numeracy.

Question 3
Belinda is on her first business trip to a foreign country. She knows that the trip is likely to be more successful if she
is aware of the countrys cultural manifestations that are hard to see "below the waterline." She observes the
following four behaviors and realizes that only one is an example of culture "below the waterline." Which one is it?

Correct

Good choice. You answered:Danielle believes that women should not raise their voices in public.

Question 4

Roberto, a senior manager with a Brazilian company, is looking for a junior executive to help him negotiate a deal in
Russia. He knows Kwame has been to Russia before, but he wants to make sure Kwame has sufficient cultural
intelligence for the job. Which of the following is not a strong indicator that Kwame has good cultural intelligence?

The correct choice is: He does not allow himself to make mistakes when collaborating with people from Russia.

Question 5

A company in Alaska is working on a collaborative project with three different fabricating plants located in Italy,
France, and Spain. Only one of the Alaskan managers has taken an action that will help to facilitate trust with their
foreign collaborators. Who was it?

Correct

Good choice. You answered: Lasha decided that the use of a Web-based program to document production
problems would make everyone accountable.

Participants should be able to document actions taken and lessons learned. Trust increases when everyone can see
that others are doing their job.

REVIEW MATERIAL
Learn > Establishing Trust > Establish shared goals and document progress

Question 6

Clarence is part of a team putting together an international mining consortium. He will have to negotiate with
partners in three different parts of the world and wants to avoid cultural misunderstandings. Which of the following
strategies is most likely to work best in all the cultures involved? Clarence should ...

Correct

Good choice. You answered: Be willing to change his negotiating style to suit the circumstances.

Clarence should use what he's learned about his counterparts and how negotiations are handled in their countries
to adapt his approach to the deal at hand. For example, if the other negotiators require consensus within their
team, Clarence should accept that the deal is going to take time.
REVIEW MATERIAL
Learn > Negotiating Across Cultures > Differences in negotiating approaches

Question 7

A Canadian health products company urgently needs to find a new distributor in Asia. The company's president is
interested in a firm called Distri-East. He sends Maria to negotiate and tells her to try to get a distribution
agreement as quickly as possible. Which of the following should Maria do first when she begins talks with Distri-
East?

Correct

Good choice. You answered: Ask Distri-Easts negotiators questions about what life is like in their country.

Negotiators in many cultures would want to get to know Maria and assure themselves that she is reliable before
doing business with her. To gain their trust she should ask them about what things are like in their country. She
should find something she has in common such as a favorite sport, wine, hobby, or piece of music.

REVIEW MATERIAL
Learn > Negotiating Across Cultures > Build trust

Question 8

During a strategy conference with overseas branch managers, Raj, a senior executive, notices that Boris and Yuri
are saying little. At one point, the two begin a quiet side conversation in their native language. Raj realizes they are
uncomfortable using the common business language. He wants to help prevent language barriers from sabotaging
the meeting. What would be the wrong thing to do?

Correct

Good choice. You answered: Tell Boris and Yuri that he understands that they are having problems with the official
language but encourage them to use it.

Raj should avoid implying that non-native speakers of the official language have a "problem" and that native
speakers do not. Instead, he should send the message that everyone shares in the challenges of cross-lingual
communication and that all participants should invest time in learning how to work and speak together.

REVIEW MATERIAL
Learn > Overcoming Language Barriers > Foster inclusive communication

Question 9

Akis company is the lead partner in an international collaboration developing home entertainment systems. The
partners have set up Web-based conferencing tools and frequently hold online meetings. As lead partner, what else
should Aki do to overcome the challenges of the physical distance between collaborators? Aki should ...

Correct
Good choice. You answered: Post each participant's personal interests on a virtual "wall" in a shared workspace to
encourage collaboration.

Some shared workspaces have several "walls" devoted to particular aspects of the collaboration. Aki could use one
"wall" to post individual collaborators' contact information, accomplishments, areas of expertise, and interests.

REVIEW MATERIAL
Learn > Surmounting Challenges of Physical Distance > Tackle the challenges

Question 10

An international bank is planning to launch a new retail product in several countries at once. As project manager,
Aileens biggest problem is aligning her global team. Which of the following steps would most effectively help align
Aileens team members? Aileen should ...

Incorrect

Not the correct choice. You answered: Establish the largest teams in locations with the most employees.

Aileen should keep team sizes relatively consistent and lead meetings from different sub-groups' home bases. This
will foster a sense of shared identity.

The correct choice is: Work with her team to develop a mission statement.

Time Management:-

Question 1

You are a time management consultant who conducts training courses for companies in your city. Your goal is to
show employees how they can achieve greater levels of productivity simply by using their time more effectively.
Which of the following advice would not appear in your course because it would be unlikely to further this goal?

Correct

Good choice. You answered: Respond to e-mails as soon as possible after receiving them.

While e-mail is an important communication tool, it can often be a significant time-waster. Answering e-mails can
make you feel as if you are working hard, when the reality is that you are doing nothing to further your high-priority
objectives. Set aside specific times during the day for responding to e-mails and be sure that the time you devote to
e-mail supports your company's objectives.

REVIEW MATERIAL
Learn > Why Manage Your Time? > Discipline and practice
Question 2

You are a learning services manager whose key responsibility is to identify and hire vendors to deliver sales training
in your organization. You have a critical goal of improving the quality of the training. What is your best time-
management strategy?

Correct

Good choice. You answered: Interview new training vendors yourself; have others manage contract renewals.

Your plan to interview training vendors yourself and delegate contract renewals is a good time-management
strategy. It is aligned with your critical goal of improving the quality of training.

REVIEW MATERIAL
Learn > Identifying and Prioritizing Goals > Align your goals

Question 3

As human resources manager for Jalna Hotel, you ask your assistant to schedule the goals and tasks required to
recruit and train staff in time for the opening of your newest Miami hotel. You show him the procedure for breaking
goals into manageable tasks. When you review the schedule he's created, you notice he has made a mistake. What
could have been his mistake? He ...

Correct

Good choice. You answered: Prioritized goals according to the amount of resources needed to complete each task.

While there is a step in the procedure where you prioritize goals and tasks, they should be prioritized in order of
value and importance. Top priority is assigned to goals and tasks having high value and primary concern. Lowest
priority is assigned to those goals and tasks having little value and little importance.

REVIEW MATERIAL
Learn > Breaking Goals into Tasks > Step 2: Prioritize tasks

Question 4

Freda is a head physiotherapist in a large, regional medical center. Each day she struggles to keep up with her
increasing patient load, leaving her stressed and exhausted. She decides to log her time for a week to see where
her time goes and what changes she can make. When Freda reviews her log, she notes the following patterns of
time use. Where is she most likely wasting time?

Correct

Good choice. You answered: From 9:00 to 9:30, she photocopies and assembles patient education materials that
she'll use during the day.
Photocopying materials is not a good use of Freda's time. It doesn't match her most important objectives and could
be easily delegated to others.

REVIEW MATERIAL
Learn > Analyzing How You Spend Your Time > Identify ways to improve your use of time

Question 5

Your executive coach shadows you through a typical workweek. She observes the way you handle paperwork, deal
with tasks you dislike, and manage your meetings and travel time. She looks for ways you could more efficiently
use your time. Which of the following activities might she identify as an area for improvement?

Correct

Good choice. You answered: When you prepare your report for the monthly sales meeting a task you detest
you try to make the work more engaging for yourself. You look for colorful binders and explore zippier slide formats.

She's spotted you procrastinating! You are prolonging the planning process with incidental tasks to avoid beginning
the work itself preparing the report. If you're having trouble starting, set an arbitrary start and go from there. If
the task is unpleasant or tedious, create some motivation for yourself: plan a reward to give yourself upon
successful completion.

REVIEW MATERIAL
Learn > Recognizing and Defeating Common "Time-Wasters" > Procrastination

Question 6

Your colleague shows you the schedule he's created for your three-day planning session. You're generally satisfied
with his planning, but there's one problem. What is it? He ...

Correct

Good choice. You answered: Schedules routine reports in the morning and concludes the day with examination of
key business challenges.

The activity that requires the most creativity (the examination of key business challenges) is scheduled to occur at
a time when participants will likely have low energy. Important work or activities that need creativity and
intelligence should be scheduled at a time when participants are most alert.

REVIEW MATERIAL
Learn > Scheduling Time More Effectively > Develop your schedule

Question 7

You have recently completed a course on efficient time usage and have been attempting to integrate a schedule
into your workday. As suggested by the course leader, you are taking time today to review your progress. You find
that you have a pattern of not being able to complete tasks within the timeframes you have allotted. What is the
best single step you can take to solve the problem?
Correct

Good choice. You answered: Determine the reason behind the scheduling shortfalls.

Determining the reason(s) behind the scheduling shortfalls is the first step in dealing with your problem. Once you
understand the reason(s), you can adjust both your scheduling system and your working patterns as required to
prevent such slippages in the future.

REVIEW MATERIAL
Learn > Monitoring and Improving Your Time-Management Strategies > Check your progress

Question 8

You are the legal assistant for the litigation firm of Dover, Slunn, and Fawn. Although you enjoy your job, you
sometimes find it difficult because each of the partners in the firm has a different management style, resulting in
conflicting demands and expectations. The outcome is wasted time and a backlog of work. You are worried because
you are getting further behind. How can you deal most effectively with this situation?

Correct

Good choice. You answered: Meet with the partners to discuss work direction, reporting, and timing.

If you and your supervisor(s) haven't agreed on goals, connected them to time frames, and ensured that your goals
support the company's strategy, you will not be able to define a coherent set of tasks or create a schedule for them.

REVIEW MATERIAL
Learn > Dealing with Time-Wasting Bosses > Clarify your goals with your boss

Question 9

Anne Higher is an aeronautical engineer and owner of the Higher Plane Assembly Plant. She frequently finds herself
becoming so involved that she works long hours and many weekends. Lately she has noticed that she has less and
less time for community activities and outings with friends. She is feeling dissatisfied with this imbalance between
her work life and her personal life. What is the first thing she should do to correct this imbalance?

Correct

Good choice. You answered: Clarify and prioritize her highest personal goals.

Clarifying and prioritizing your personal goals is the first step in balancing your work and home life. Clarify your
goals and define them in specific terms. Then prioritize your goals, identify the tasks you must complete to achieve
your highest goals, and schedule the related tasks accordingly. Finally, seek opportunities to free up personal time
by evaluating your relationships and commitments, as well as reducing clutter.

REVIEW MATERIAL
Learn > Balancing the Demands on Your Work and Personal Time > Take control of your personal time

Question 10
As a supervisor in your company you realize the importance of helping your employees manage their time. To
achieve this goal you draw up a list of four strategies employees could try. Which is the least effective strategy?

Incorrect

Not the correct choice. You answered: Use flexible schedules to maximize employee efficiency.

This would likely be a helpful strategy because it allows employees to schedule their working time in a manner that
facilitates their other obligations, such as driving children to school.

The correct choice is: Ensure that work is done in the office to maximize the use of equipment and resource time.

Difficult Interactions:-

Question 1

Your colleague Mike has just been promoted to director of business services. He tells you that he is having difficulty
with Faye, one of his new direct reports. Each time he assigns a task to her, she reacts negatively and complains
that she already has too much to do. Mike doesn't think Faye is overloaded; he thinks she is just a chronic
complainer. What is the best advice you could give Mike to help him more effectively manage his interaction with
Faye?

Correct
Good choice. You answered: Before doing anything, decide whether the problem is worth addressing.

Mike should evaluate whether the risks of the difficult interaction are worth the benefits of an improved situation. If
they are, he should map out a plan and carry it out.

REVIEW MATERIAL
Learn > What Are Difficult Interactions? > Overcome barriers to action

Question 2

The work of your product development team has come to a halt. Jack voices his opinion that the new cereal you are
developing should be targeted to weight-conscious teenagers. Patti argues that health-conscious parents are a
better target market. The statements that Jack and Patti have made describe their ...

Correct

Good choice. You answered: Positions.

The statements that Jack and Patti have made reflect their positions the stances they are taking on the target
markets. Jack's position is that the cereal should target weight-conscious teenagers; Patti's position is that it should
target health-conscious parents.

REVIEW MATERIAL
Learn > Causes of Difficult Interactions > Differences in position and interests

Question 3

You manage a team responsible for selecting new locations for Go-Food-Go, an international fast-food restaurant
chain. While selecting a location in one region, your team reaches a stalemate. Kim says the new restaurant should
be located on the major highway at the edge of town. Randy says it should be at a busy intersection in town. Where
are Kim and Randy likely to have the most difficulty when trying to reconcile their differences?

Correct

Good choice. You answered: Understanding the underlying interests.

Kim and Randy may not be aware of the underlying interests that shape each of their positions. To uncover these
interests, you need to ask open-ended questions.

REVIEW MATERIAL
Learn > Causes of Difficult Interactions > Differences in position and interests

Question 4

Rodney, one of your direct reports, has been causing trouble since coming to your department. He starts rumors
about other members of the team and tries to isolate certain people, leaving them out of important discussions and
team events. You talk to Rodney's former manager and find out he behaved the same way there, despite the
managers repeated efforts over several months to solve the problem. What course of action will you most likely
have to take?

Correct

Good choice. You answered: Give Rodney a deadline for improving his behavior and dismiss him if he doesnt
change.

When someone has destructive relationships with many people across a wide range of situations, you will need to
take action such as formally disciplining the employee and letting him go if disciplinary action doesnt lead to
improvement.

REVIEW MATERIAL
Learn > To Manage a Difficult InteractionOr Let It Go? > Consider the potential for improvement

Question 5

Billy, a food chemist at Ice Castle Ice Cream, is on a committee developing a novelty ice cream flavor, an
assignment that will last about two weeks. He gets along well with the other five committee members, except Herb,
the sales manager. Billy doesn't often work with sales people and he wonders if theyre all like Herb: loud,
obnoxious and rude. He also wonders whether he should confront Herb about his behavior. What would be bad
advice to give Billy?

Correct

Good choice. You answered: Solve the problem any way possible, because all relationships within the company are
important to its success.

This would be bad advice to give. Billy should consider how important his relationship is with Herb and whether the
relationship is short term or long term. Since Billy doesn't often work with sales people, his relationship with Herb is
likely not very important. Also, the relationship will be short-term about two weeks. Billy may decide it's not
worth the time and energy needed to improve the relationship.

REVIEW MATERIAL
Learn > To Manage a Difficult InteractionOr Let It Go? > Identify important relationships

Question 6

Juan is in charge of safety testing in your department. He doesnt seem to be keeping on top of his work and
appears resentful when you speak to him about it. The project is now two weeks behind. You wonder if your
management style might be part of the problem, so you ask a consultant how to handle the situation. What is the
best advice she can give you?

Correct

Good choice. You answered: Tell Juan what happened when the safety testing didnt get finished on time at the last
company where you worked.
By sharing the experiences, as well as the information and assumptions behind your view of the situation, you and
Juan will begin understanding each other which is essential to resolving the problem.

REVIEW MATERIAL
Learn > Assessing the Facts > Share and explain your impressions of what's going on

Question 7

The sales vice president calls Malik and Joel into his office so they can explain why second-quarter sales were down.
During the meeting, Joel complains about quality problems with a new supplier Malik has chosen. That angers Malik
because Joel has never talked to him about quality problems before. After the meeting, he decides to talk to Joel
about it. What is the best way for Malik to express his anger?

Correct

Good choice. You answered: When you talked about quality problems in front of the boss, I felt embarrassed, and
that made me angry."

After rethinking destructive emotions, Malik should describe the feelings hes experiencing as a result of the difficult
situation. Maliks goal is to express emotions without judging or blaming the other person.

REVIEW MATERIAL
Learn > Identifying the Emotions > Express your emotions

Question 8

Maria, a manager in another department, is having personality clashes with the newest member of her team. She
admits to you that the confrontations are beginning to make her doubt her abilities as a manager. She asks you to
suggest a strategy to help her protect her self-image. What is the best suggestion you can give her?

Incorrect

Not the correct choice. You answered: When you feel your self-image being threatened, concentrate your
thoughts on other issues.

This is not a good suggestion. Burying feelings and resorting to a detached manner does not enable a person to
listen to negative feedback and make the changes needed to improve interactions with others.

The correct choice is: Write down what you consider to be your qualities as a manager.

List all the assumptions that influence your self-image. Ask yourself which of these assumptions evoke the strongest
feelings. These are the assumptions that will most likely trigger a feeling of threat to your self-image if theyre
called into question during a difficult conversation.

REVIEW MATERIAL
Learn > Dealing with Threats to Your Self-Image > Handle threats to your self-image

Question 9
You are a manager in the product development department of a nation-wide handyman store. While discussing the
rollout of a new tool in a monthly meeting, your direct report Franois vehemently opposes the suggestions of
several team members. He appears hostile and is making other team members uncomfortable. What is the least
productive way to frame the problem when speaking to Franois?

Incorrect

Not the correct choice. You answered: Perhaps I might have contributed to this standoff by not making my
selection criteria clear enough.

This is actually a productive way to frame the problem. Emphasize contributions to the problem, not blame.

The correct choice is: Franois, theres no place in this group for somebody who doesnt take the selection
process seriously.

This is not a productive way to frame the problem. Focus on perceptions, not presumed truths. In this case, you are
presuming that Franois does not take the selection process seriously, but he may have other reasons.

REVIEW MATERIAL
Learn > Solving the Problem > Continue framing the problem productively

Question 10

Youve been asked to teach a night school course on conflict resolution. Tonight, the topic is conflict in the
workplace. Which of the following is most likely to appear in your lecture notes?

Incorrect

Not the correct choice. You answered: Even if a dispute does not interfere with employee performance, disrupt
the workplace, or violate policy, failing to intervene will make you look like a weak manager.

This would not appear in your lecture notes. Many experts believe that in these circumstances, a policy of "benign
neglect" may be you best approach.

The correct choice is: When intervening, a manager should let the employees do most of the work in resolving
the situation.

Feedback Essentials:-

Question 1
Which of the following statements about giving and receiving feedback is not true?

Correct

Good choice. You answered: Giving feedback is an opportunity to offer judgments on your boss or your peers.

Feedback should not be a judgment about another person. Feedback is intended to be an objective message about
behavior and its consequences, either to recognize a job well done or to suggest how to improve performance.

REVIEW MATERIAL
Learn > What Is Feedback? > Recognize the importance of feedback

Question 2

In which of the following situations would feedback be least appropriate?

Correct

Good choice. You answered: Maria checks her calendar and decides it is time to review Georges job performance
over the past six months.

This situation calls for performance evaluation, not feedback. It is important to realize that a feedback session has a
different purpose than a performance evaluation meeting. Formal performance appraisals are conducted at regular
intervals (usually every six months or at least once a year). The focus of a performance evaluation is to review an
employees past work whereas feedback is given on an as-needed basis and looks to the future to determine how
that employee can grow.

REVIEW MATERIAL
Learn > Deciding When and How to Deliver Feedback > Feedback sessions versus performance evaluations

Question 3

You are the sales manager of a Japanese auto parts manufacturer. You have heard rumors that Brian, your North
American sales representative, is impatient with potential customers. You suspect that his annoyance is negatively
affecting his sales performance and therefore the company's North American sales overall. At a recent convention,
you observe Brian's impatience with prospects firsthand. What is your best approach for providing feedback?

Correct

Good choice. You answered: Focus your comments on what Brian could do to control his impatience next time.

Focusing your comments on an issue that Brian can work on in the future, such as controlling his impatience with
customers, is effective feedback. When giving feedback, make sure to focus on improving performance dont use
feedback simply to criticize.

REVIEW MATERIAL
Learn > Giving Feedback Effectively > Guidelines for effective feedback
Question 4

A colleague arranges lunch to provide feedback on the marketing collateral you recently developed. He tells you
that the materials are poorly designed and lack creativity. You restate his comments in different words and then ask
him to explain in greater detail. You assess his emotions and notice that his body language signals impatience. You
decide that he is trying to be helpful, but is annoyed because he feels the materials reflect badly on the company.
What did you fail to do to get the most from his feedback?

Correct

Good choice. You answered: Separate fact from opinion.

When receiving feedback, you should be prepared to separate fact from opinion. It is only your colleagues opinion
that the marketing materials are poorly designed. Its not that opinions should be discounted, but they dont always
carry the same weight as established facts. To get the most out of your colleagues feedback, you need to establish
what specific aspects of the design are indeed poor and need improvement.

REVIEW MATERIAL
Learn > Receiving Feedback Openly > Tips for staying open during the feedback session

Question 5

Sam, if you have five minutes, Id appreciate your advice. Folks are really tense about the pending merger and it
has started to affect production. Thats not what anybody wants. I want to meet with the production division about
the issue. Knowing how popular you are in the plant, do you think the meeting is a good idea? This feedback
approach is designed for what type of person?

Correct

Good choice. You answered: Relater.

This is appropriate feedback for a relater. Sam responds to an emphasis on how his feedback will help to create a
more stable and productive work environment. Give relaters information and ask them for their opinion.

REVIEW MATERIAL
Learn > Customizing Feedback > Behavioral typology

Question 6

One of your direct reports, Anna, redesigned the production timetable at your plant to facilitate faster overseas
deliveries. The plan still needs some work. Knowing your direct report thrives on authority and control, what is the
best way to deliver feedback?

Correct

Good choice. You answered: Describe the areas in the plan that you think need work and ask Anna what she thinks
should happen next.
Since Anna is a director someone who seeks to establish authority and control describing the areas of the plan
that need work and soliciting her opinion is the most appropriate way to deliver feedback. When giving feedback to
directors, the best way to work is to address the issues directly and give them control during the feedback process.

REVIEW MATERIAL
Learn > Customizing Feedback > Behavioral typology

Question 7

You have always encouraged your team to be open to giving and receiving feedback. You notice, however, that one
of your employees is unable to give or receive feedback effectively because of his attitude. Which employee is it?

Incorrect

Not the correct choice. You answered: Faud, who tries to maintain his sense of self-worth when your comments
are negative.

Fauds attitude enables him to receive feedback effectively. He doesnt allow suggestions for improvement to
diminish his self-worth.

The correct choice is: Andre, who declines personal recognition awards, insisting that the awards be shared with
his team.

Andres attitude is a barrier to receiving feedback effectively. He is uncomfortable receiving affirming feedback
personal recognition awards because he does not want to be set apart from others.

REVIEW MATERIAL
Learn > Managing Barriers to Feedback > Barriers to receiving feedback

Question 8

Which one of the following statements is true?

Correct

Good choice. You answered: Feedback needs to address behavior that the receiver can actually change.

Certain types of change, such as those involving personality characteristics, can be too difficult or impossible
for some people. Feedback should be limited to changes that are within the persons control.

REVIEW MATERIAL
Learn > Acting When Feedback Calls for Change > Affecting change

Question 9

In which of the following situations would feedback be least effective?


Correct

Good choice. You answered: Even after receiving several customer service training courses, Brigid remains ill-at-
ease when talking with clients and seems unable to be genuinely customer friendly.

Feedback is least likely to be effective in this situation Since Brigid has already received several customer service
training programs that have not changed her behavior, being customer friendly seems beyond her control.
Generally speaking, providing feedback is not recommended when a change is too difficult for the person or beyond
a persons control.

REVIEW MATERIAL
Learn > Acting When Feedback Calls for Change > Affecting change

Question 10

Troy is a market researcher for a multinational advertising firm. Colleagues have started to complain about her short
workdays and lengthy personal phone conversations. As the manager of the research division, you know that you
need to address the growing discontent within your team. You find out that a family problem is diverting Troy's
attention from work. Which of the following is the most appropriate course of action?

Correct

Good choice. You answered: Recognize that Troy is having a personal difficulty and explore ways to prevent it from
having a negative impact on her work.
New Manager Transitions:-

Question 1

Which of the following statements is a myth about managers and not a reality?

Correct

Good choice. You answered: The skills that lead to success as an individual contributor closely resemble those
needed to manage effectively.

The idea that individual contributors and managers use the same or similar skills to perform effectively is a myth.
The skills that lead to success as an individual contributor actually differ markedly from those needed to manage.
Many managers mistakenly believe that they'll be using the same skills they used as individual contributors
except that they'll be applying those skills to more challenging projects.

REVIEW MATERIAL
Learn > Myths About Managers > Myth: You'll use the same skills

Question 2

You have recently accepted the role of distribution manager for Cayuga Foods. Colin is an order-taker on your new
team. What expectation would Colin have for you, his manager? He would expect you to ...

Correct

Good choice. You answered: Create conditions that will help him succeed.

Colin would expect you to create conditions for his success by providing leadership, keeping things running
smoothly, anticipating long-term changes in the business environment, and building effective networks.

REVIEW MATERIAL
Learn > Roles and Expectations > Direct reports' expectations

Question 3

Janna has recently been promoted to manager of technology services at Expedient Airlines. One of her team's first
projects is to evaluate, select, and implement new scheduling software. In which of the following instances is Janna
not behaving like a manager? She ...

Correct
Good choice. You answered: Measures her success by whether she selects the best possible software package for
the task at hand.

Janna is not behaving like a manager if she defines her success in terms of her own performance (identifying the
best product) rather than her group's success. As a manager, Janna's primary responsibility is to get things done
through others she needs to measure success differently than she did as an individual contributor.

REVIEW MATERIAL
Learn > Taking a Broader View > Measure your success by your group's

Question 4

During a team meeting, Elliot, the team leader, notices that Linda is uncomfortable with Fred's point of view on
resource allocation. After the meeting, Elliot asks Linda why she did not raise her concerns. She explains that she
was worried that Fred would take her comments personally. Elliot then realizes that he has probably
overemphasized support within the team. What should Elliot do to correct this imbalance?

Correct

Good choice. You answered: Foster confrontation among team members by providing training in giving
constructive feedback.

With a team whose members avoid conflict, the best strategy is to find ways to encourage members to express
conflicting thoughts or ideas without letting disagreements become too personal. Training in how to give
constructive feedback can be helpful in this case.

REVIEW MATERIAL
Learn > Managing Teams > How to manage a team

Question 5

Celia is an experienced insurance group policy administrator on your team. She never lets problems in the team get
her down and she performs her role with accuracy and a high level of efficiency. When you ask her if she would like
to mentor a new member of the team, she states that she probably couldnt handle the responsibility. What would
be the best managerial style to adopt with Celia?

Correct

Good choice. You answered: Supportive.

When an employee appears reluctant to fully engage in the work at hand, you need to adopt a supportive style that
encourages the person to gradually build on existing strengths in order to take more risks.

REVIEW MATERIAL
Learn > Managing Individuals > Managerial styles

Question 6
You manage a team of account managers at Superior Performance Consulting. Which of the following scenarios is
the best example of a coaching opportunity?

Incorrect

Not the correct choice. You answered: You want to advise Pina that youre uncomfortable with the way she
speaks to you in meetings.

Coaching is not a way to address interpersonal issues or express criticism. Coaching involves working together to
improve someones performance on the job.

The correct choice is: Tony has set goals for acquiring new clients that are unrealistic.

Coaching is a good way to help direct reports clarify and work toward performance goals. If Tonys goals are
unrealistic, coaching can help him learn to set more realistic ones.

REVIEW MATERIAL
Learn > Managing Individuals > Coaching

Question 7

Greg, one of your colleagues, is having trouble motivating his team. This affects you because your team works
closely with his. You say, "Greg, I've noticed that the way you assign work to your team isn't motivating them to
achieve." Greg appears annoyed and ignores your feedback. What could you do differently next time to avoid this
outcome?

Incorrect

Not the correct choice. You answered:Focus on Greg's results and avoid talking about how he gets his work done.

It is appropriate to provide feedback on Gregs process how he gets his work done.

The correct choice is: Offer specific advice on how he could improve the work-assignment process.

Offering Greg non-critical advice on ways to improve his work-assignment process would be more useful than
enumerating his faults. Effective feedback can help the recipient improve not only the way the person gets his work
done (process) but also how the individual performs on measurable on-the-job achievements (results) and how well
the person interacts with others (relationships).

REVIEW MATERIAL
Learn > Managing Individuals > Why give feedback?

Question 8

Tom has just become a corporate accounts manager for a large consulting firm. He wants to ensure the health,
happiness, and productivity of his team. To ensure this, which, if any, of the relationships needs to be stronger than
the others?
Correct

Good choice. You answered: They should all be equal.

The most effective managers attend to all three sets of relationships with equal energy. Relationships in one set
affect relationships in the other two.

REVIEW MATERIAL
Learn > Putting It All Together > The "Triangle of Relationships"

Question 9

Which of the following statements about diversity is not true?

Correct

Good choice. You answered: Diversity means primarily racial or gender differences.

Saying that diversity means primarily racial or gender differences is not a true statement. Diversity can mean
much more than race or gender. Your supervisor, peers, and direct reports can differ in many ways, including
tenure, cultural background, physical ability, and working and learning style. People also differ along more abstract
dimensions, such as professional motivations, management preferences, experience level, and personal work style.

REVIEW MATERIAL
Learn > Promoting Diversity and Understanding Group Culture > Forms of diversity

Question 10

As the new regional manager for an office services chain, you spend the first week talking to each store manager in
your region to learn how the group solves problems, communicates, and interacts. You discover that more than half
of the store managers are following outdated promotional strategies. You immediately replace the old policies in all
stores with new policies based on industry best practices. You are surprised to meet resistance to this change. What
went wrong?

Correct

Good choice. You answered: You made major changes too soon.

The reason you met resistance is probably that you tried to implement change too soon. It is important to spend
time learning about the culture you have inherited before attempting to redefine processes and policies.

REVIEW MATERIAL
Learn > Promoting Diversity and Understanding Group Culture > Understand group culture

Question 11
You are the director of human resources at Hiroma Electronics. You recently hired Ian as your training manager. It is
his first time managing others, and he is concerned that he doesn't have what it takes to be a successful manager.
To help him with self-assessment, you recommend that he ...

Correct

Good choice. You answered: Gauge his impact on the organization by evaluating his influence on his training team.

Determining how he is exerting an impact on the organization through others (for example, his direct reports) will
help Ian gauge his success as a manager.

REVIEW MATERIAL
Learn > Building Self-Awareness > Assess your ability to do the job

Question 12

Phoebe is production supervisor at a large manufacturer. Upon reviewing her team's weekly timecards, she
discovers that Darren has been late on three separate occasions this week, in spite of receiving several prior
warnings about the importance of promptness to line operations. Phoebe feels frustrated and wants to walk out to
the manufacturing floor to confront Darren. She waits until her frustration subsides, and then asks Darren to remain
after his shift to discuss compliance with shift procedures. Which emotional intelligence skill has Phoebe
demonstrated?

Correct

Good choice. You answered: Self-regulation.

Phoebe is demonstrating self-regulation. She controls her impulse to express her frustration in front of the
manufacturing line and channels her emotion for the purpose of coaching Darren to achieve better performance.

REVIEW MATERIAL
Learn > The Power of Emotional Intelligence > Components of emotional intelligence

Question 13

Doug has recently become delivery systems manager at a major transportation company. When a friend asks how
he is enjoying his new role, Doug admits that he is suffering from role strain. Which of the following examples best
illustrates Doug's situation?

Correct

Good choice. You answered: Doug is overwhelmed by his work and feels that he is accountable to everyone.

Doug is likely feeling that he has too much work and that he has to answer to too many people. Both of these
feelings are associated with role strain a form of stress that derives from the overload, ambiguity, and conflict
woven into the managerial role.
REVIEW MATERIAL
Learn > Coping with New Emotions > Role strain

Question 14

You promote your top salesperson, Mario, to sales manager. In your first meeting after his promotion, he describes
the stressful emotions he is experiencing in becoming a manager. You tell Mario that all the emotions he's described
are to be expected, but one in particular is especially difficult for most new managers to tolerate. Which is it?

Correct

Good choice. You answered: Fear of failure.

Typically, new managers have the least tolerance for performance anxiety, or fear of failure. Many managers have a
long history as high achievers in their individual contributor role. As a result, they find it especially difficult to
imagine coping with the feelings of shame and guilt that accompanies failure.

REVIEW MATERIAL
Learn > Coping with New Emotions > Stressful emotions of transformation

Question 15

Ira is the new advertising manager for Excel Travel Services. Before taking this job, Ira was a part-time actor who
had appeared in some television commercials and local theater. His first task as new manager is to promote Excel's
Caribbean cruises. His supervisor advises him to use a collaborative process to accomplish his task. Which of the
following is not an example of a collaborative process?

Correct

Good choice. You answered: Ira leverages his media experience by making local appearances to build awareness
of the cruises.
Persuading Others:-

Question 1

Which of the following approaches would not be used by a skilled persuader?

Correct

Good choice. You answered: Adopt a firm position on the issue and stick to it.

An approach that is not effective in persuasion is sticking rigidly to your position. Rather, persuasion is a complex
process that requires back-and-forth dialogue and a mutual process of learning and negotiating. Skilled persuaders
frame goals on common grounds, and describe the benefits of the position theyre advocating in terms of what both
they and their listeners value.

REVIEW MATERIAL
Learn > Persuasion Overview > The elements of persuasion

Question 2

Trust plays a large role in a persuader's ability to build personal credibility. Which of the following individuals is
using a strategy that will not enable him or her to earn others' trust?

Correct

Good choice. You answered: To build and enhance the status of his department, John uses his influence and
warmth to persuade his best performers not to seek promotions to other departments.

One sure way not to build trust is to put your own best interests over those of others for example, by hoarding
top performers in your department. Youll earn a lot more of your colleagues trust by showing that you have their
best interests at heart; for instance, by being willing to lose top performers to other departments if thats whats
needed to help people develop their professional skills.
REVIEW MATERIAL
Learn > Building Your Credibility > Trust

Question 3

Which of the following situations illustrates the best example of an individual building his or her expertise?

Correct

Good choice. You answered: Eleni insists on small-scale testing new ingredient combinations before proposing the
introduction of a new brand of beer to senior management at Nova Brewery.

In addition to researching your ideas, letting people know about appropriate credentials, and sharing accolades and
testimonials, launching small pilot projects can help you strengthen your expertise. Such projects demonstrate that
your ideas deserve serious consideration, and help generate firsthand information about their benefits.

REVIEW MATERIAL
Learn > Building Your Credibility > Expertise

Question 4

In which of the following examples would you be directing your persuasive efforts toward influencers?

Correct

Good choice. You answered: As head of product development at Harvest Foods, you want to adopt new software
that you believe will enable your department to design better products. You contact the heads of information
technology, purchasing, and finance to discuss your idea.

Influencers are individuals who participate in the decision-making process by providing advice and information to
key stakeholders and decision makers. A product development manager proposing new software would benefit from
getting the opinions of the heads of information technology, purchasing, and finance all of whom might influence
the individuals who ultimately will decide whether to approve adoption of the new software.

REVIEW MATERIAL
Learn > Understanding Your Audience > Identify decision makers, key stakeholders, and influencers

Question 5

You are presenting a proposal to senior management at Swift Athletics. Your goal is to persuade senior managers to
approve your proposed reorganization of the companys running-shoe product line. The plan includes the phasing
out of the companies long-time trademark shoe. You expect a largely hostile reaction and plan to tailor your
presentation accordingly. Which persuasion strategy would be most effective?

Correct
Good choice. You answered: You emphasize aspects of the proposal that your listeners do support.

Focusing on areas you and your audience do agree on is appropriate for a hostile audience in which the group
overall disagrees with you. By focusing on areas of agreement, you can stress the win-win outcome youre looking
for.

REVIEW MATERIAL
Learn > Understanding Your Audience > Categories of receptivity

Question 6

Phoebe is head of operations at Acme Tool and Hardwares North American call center. Her direct reports are under
stress so she meets with her department head to ask for more staff. Since her supervisor is new to the department,
Phoebe does some homework to find out his decision-making style. She determines that his initial reactions to new
ideas typically mislead people into thinking he is very enthusiastic and that hell surely support the ideas. What
type of decision-making style best describes Phoebes supervisor?

Correct

Good choice. You answered: Charismatic.

Phoebes supervisor seems to be demonstrating the characteristics of a charismatic decision maker. His initial
enthusiasm may mislead her into thinking she has scored an immediate success. However, charismatic decision
makers ultimately base their decisions on balanced information.

REVIEW MATERIAL
Learn > Understanding Your Audience > Decision-making styles

Question 7

As public relations manager for the city council, you have been asked to speak with local citizen groups to garner
support for a series of complex rezoning and traffic rerouting measures. You suspect that many of the local citizen
groups will have little in-depth knowledge about the citys zoning problems and will not have decided whether to be
supportive. If your instincts were accurate, what type of presentation structure would you use with this audience?

Correct

Good choice. You answered: Problem-solution.

When you want to describe a complex solution to an audience that might be uninformed about the problem, use the
problem-solution structure. To present your case to this audience, first describe the pressing problem and then solve
it by offering a convincing solution.

REVIEW MATERIAL
Learn > Winning Your Audience's Mind > Structure your presentation effectively

Question 8
Alexandra is a manager in a fashion-design company. She is meeting with the chief executive officer of a Milan
textile firm to discuss a possible partnership agreement. So far, the CEO has rejected Alexandras suggestions for
structuring a partnership. Alexandra begins the meeting by suggesting that the textile company will be sailing
against the wind by trying to break into the North American fashion scene without a good partner. She outlines how
her company could help and suggests that by working together they can chart a successful course to establish an
effective market presence. Which persuasion device is Alexandra using?

Incorrect

Not the correct choice. You answered: Analogies

Alexandra has not used an analogy a like or as comparison that enables someone to relate a new idea to one
thats already familiar to an audience.

The correct choice is: Metaphors.

Alexandra has used a sailing metaphor to communicate the difficulties her potential partner may face, evoke a
vision of the better future in store if the two companies strike a deal, and persuade her potential partner to consider
a different outlook.

REVIEW MATERIAL
Learn > Winning Your Audience's Heart > Metaphors

Question 9

Which of the following statements about emotion-evoking presentations is true? Emotion

Correct

Good choice. You answered: Usually plays a more powerful role in human decision making than logic.

Emotion-arousing proposals tend to prompt behavioral changes more quickly than logical appeals do because
theyre compelling and easy for audiences to process. Such proposals also turn listeners attention away from the
fact that the presenter is seeking to persuade them.

REVIEW MATERIAL
Learn > Winning Your Audience's Heart > Appeal to emotions

Question 10

Jordan is chief executive officer of an international airline. In seeking to turn around the airlines bleak financial
outlook, he wants to secure wage rollbacks from all the unions. The most powerful and influential union represents
the airlines mechanics. Jordan secures this unions support and uses their agreement to win support from the other
unions. What persuasion trigger did Jordan use?

Correct
Good choice. You answered: Social proof.

Jordan used the social-proof persuasion trigger by gaining the support of a powerful union. Because the other
unions identify themselves strongly with the first union, they were more willing to follow its lead.

REVIEW MATERIAL
Learn > Understanding Persuasion Triggers > Social proof

Question 11

Diederick is head of North American sales for a company that markets a gene-based test for cancer. He has been
invited to speak at a session on medical errors at a convention of medical doctors. He begins his presentation by
saying: How does it feel to know that one in three of you will be sued for malpractice for a misdiagnosis that could
have been avoided? Diedericks approach exemplifies a:

Correct

Good choice. You answered: Disturbing question.

Diederick asked a disturbing question to get at the heart of his listeners greatest concerns or problems. The
question will make the solution he proposes more attractive, and make his listeners more willing to pay a premium
to solve their problem.

REVIEW MATERIAL
Learn > Leveraging the Power of Audience Self-Persuasion > Questioning

Question 12

Which of the following strategies is not an example of active listening?

Correct

Good choice. You answered: Ask rhetorical questions.


Managing Essentials:-
Question 1

Paul is studying marketing at his local community college. His current topic covers the various types of
needs that can be met by a marketing campaign. Unfortunately, his study material has an erroneous
statement in it. Which statement is incorrect?

Correct

Good choice. You answered: Secret needs are those which customers will only buy secretly, on-line.

This is an incorrect definition. Secret needs are actually consumer needs that customers will not openly
acknowledge as their real reason for purchasing a product. For example, someone may buy a certain
type of car because it fulfills a strong need for social status although they will not admit this fact. .

REVIEW MATERIAL

Learn > Satisfying Your Customers > Customer needs

Question 2

Which form of customer demand is described as "A strong need that can't be satisfied by existing
offerings?"

Correct

Good choice. You answered: Latent demand.

"Latent demand" best describes a strong need that can't be satisfied by existing offerings.
REVIEW MATERIAL

Learn > Developing Your Marketing Orientation > Manage demand

Question 3

Four different conceptual approaches to marketing are defined below. One of these definitions is
incorrect. Which one is it?

Correct

Good choice. You answered: Societal marketing relies on social networking to sell products.

This definition is incorrect. In societal marketing, a companys task is to determine its target
customers needs and to satisfy them in ways that preserve or enhance the well being of both
customers and society. The focus is on building social and ethical considerations into marketing
practices and on balancing profits with both customer satisfaction and public interest.

REVIEW MATERIAL

Learn > Developing Your Marketing Orientation > Five approaches to marketing

Question 4

Ashanti is a marketing executive for a large financial services company. She is creating a marketing
plan for a new savings product. She wants to convey the message that people who buy the product are
very intelligent and thoughtful. Which one of the forces that affect consumer buying is she focused on?

Correct

Good choice. You answered: Personal forces.

Ashanti considered personal forces in designing her plan. By giving the message that people who buy
the product are intelligent and thoughtful, she is appealing to customers self-image, which is one of
the personal forces affecting consumer buying.

REVIEW MATERIAL

Learn > Understanding Your Consumer Market > Forces that affect consumer buying

Question 5

You have been invited to give a guest lecture at a local college on how companies market to other
companies and organizations as opposed to individual consumers. Which of the following statements in
your lecture is incorrect?

Incorrect

Not the correct choice. You answered: Consider attitudes about how to do business in the region where
the buying organization operates.

You actually would want to make this statement during your lecture. Such attitudes are examples of
the cultural forces that can influence how organizations make purchasing decisions.

The correct choice is: Consider the conscious and subconscious needs pushing the buyer to make a
purchase.
You would likely not want to make a statement about conscious and subconscious needs during your
lecture. These are psychological factors that exert a greater influence on individual consumers than on
organizations. Marketing to businesses requires very different strategies than marketing to individuals.

REVIEW MATERIAL

Learn > Understanding Your Organizational Market > Influences on purchasing decisions

Question 6

Toy Joys Inc. manufactures a range of toys including Cindy Chatters, a talking doll. The doll is the
companys biggest seller, and the market has attracted several competitors. Which of the following
most likely presents the biggest competitive threat to the Cindy Chatters doll?

Correct

Good choice. You answered: A newly formed toy company that is considering launching a series of
dolls.

Companies are most likely to be hurt by their potential and emerging competitors than by existing
rivals. Emerging rivals havent yet declared themselves as players in the industry, so its harder to
analyze their strategies, strengths and weaknesses, and ways of doing business.

REVIEW MATERIAL

Learn > Understanding Your Competition > Identify competitors

Question 7

Andrea is a marketing manager for FunFone Inc., a manufacturer of cellular phones. The company is
about to launch its latest digital model, the D-20. It has decided to base its marketing campaign strictly
on product positioning. Which of the following marketing strategies would Andrea use for product
positioning? Andrea would ...

Correct

Good choice. You answered: Advertise the D-20 as the worlds only fail-safe cell phone.

Product positioning means communicating the central benefit of a product. FunFone would be targeting
buyers for whom reliability is a major concern if they advertised that the D-20 is the worlds only fail-
safe cell phone.

REVIEW MATERIAL

Learn > Developing Your Marketing Strategy > Positioning and branding

Question 8

Managers decide to examine Air-Tones marketing plan for its line of home entertainment equipment.
They decide that their marketing objective should be to maximize profit while maintaining current
market share. They also decide to create and heavily advertise some new product lines. At what point
in its life cycle is the Air-Tone product line?
Correct

Good choice. You answered: Product maturity.

Companies that are marketing mature product lines often emphasize maximization of profits and
defense of market share. They seek to diversify their brands and energetically promote new lines to
encourage product switching.

REVIEW MATERIAL

Learn > Developing Your Marketing Strategy > Strategies for product life cycles

Question 9

Aazim is the marketing manager for a medical devices company. He decides to use push marketing
to promote a new walker for elderly people. Which of the following would not be included in his
campaign?

Correct

Good choice. You answered: A series of late-night television commercials about the advantages of the
product.

Aazim would not include late-night television commercials in his campaign. Advertising is one of the
most powerful forms of pull marketing, not push marketing. Ads are a paid form of impersonal
promotion that can appear in many venues, such as television, print brochures, billboards, and Web
sites.

REVIEW MATERIAL

Learn > Marketing Communications > Push marketing

Question 10

You have presented a seminar on introductory marketing to a group of junior employees. At the end of
the seminar you ask them to write a brief essay on the topic of new-product development. Which of
the following statements represents the kind of thinking youd like to see in their essays?

Incorrect

Not the correct choice. You answered:Discussing new product ideas with key consumers before the
product is ready for rollout is dangerous. It could tip off competitors.

Companies do need to test-market products with key consumers in order to gauge whether the
products are technically and commercially sound. They also need to assess how enthusiastically
customers may embrace the products.

The correct choice is: Dont rely too heavily on your customers loyalty it can change quickly. You
never know when customers will decide to abandon your product.

The seminar participants need to understand that customers attitudes toward existing products can
change quickly and unexpectedly. In most businesses, companies are under pressure to constantly
come up with fresh offerings or improvements on existing products.

REVIEW MATERIAL

Learn > New Product Development > Test your ideas


Question 11

The Tool Trolley has developed its own line of household screwdrivers. These will be sold directly to the
public through hardware and department stores. Which of the following strategies would not be used
to test-market this product?

Correct

Good choice. You answered: Beta testing.

Beta testing would not be on the list because it is primarily used for test-marketing business goods
rather than consumer products. Beta testing involves sending units out to customers and soliciting
feedback, often related to a specific list of concerns or issues.

REVIEW MATERIAL

Learn > New Product Development > Test-market to organizations

Question 12

Goodley Insurance recently unveiled a new life insurance product aimed at older consumers. The
company wants to determine whether it is pursuing the best market, product, and channel
opportunities. What is the best type of marketing control to use?

Incorrect

Not the correct choice. You answered: Profitability control.

Profitability control helps you see where your company is making and losing money. Marketing
controllers measure profitability by product, territory, customer segment, channel, and order size as
well as return on investment (ROI).

The correct choice is: Strategy control.

To figure out whether theyre pursuing the best market, product, and channel opportunities, companies
use strategy control. Top managers and marketing auditors review marketing effectiveness and the
companys social and ethical responsibilities.

REVIEW MATERIAL

Learn > From Marketing Plan to Market > Control the marketing process

Question 13

Al is the president of Come N Go Courier. He is eager to increase his direct-marketing efforts but has
some concerns about it. Which of his concerns is not valid?

Correct

Good choice. You answered: Sending coupons, reminders, or special offers has very little impact on
customers.

Direct marketing of this kind can deepen customer loyalty by sparking customers interest and
enthusiasm. Reminders or other timely offers can also reactivate customer purchases. Companies
can be successful with direct-marketing approaches if they provide honest, well-designed marketing to
those customers who appreciate being contacted.
REVIEW MATERIAL

Learn > Direct Marketing > The dark side of direct marketing

Question 14

Lighthouse Financial specializes in providing women with personalized financial planning. The company
is getting involved in online marketing and they hire you to write a report about the advantages and
disadvantages of this approach. Which of the following statements would you leave out of your report?

Incorrect

Not the correct choice. You answered: Advantage: Online consumers are likely to be discerning
shoppers who value gathering information.

You would want to include this statement in your report. Online consumers tend to be well-informed
and discerning shoppers. Therefore, it will be important for Lighthouse to clearly communicate its
value proposition.

The correct choice is: Disadvantage: Since most online consumers are men, your Web site will not
reach many women who are Lighthouses target customers.

You would want to leave out a statement saying that most online consumers are men. Currently, online
consumers comprise an almost equal number of men and women.

REVIEW MATERIAL

Learn > Direct Marketing > Characteristics of online marketing

Question 15

Mandy decides to use relationship marketing to sell her companys newest electronic organizer. She
recommends that the engineers remove some problem features. She conducts market research on the
prototype with potential customers in their offices. She develops a detailed database of personal
customer information for future product launches. She also does a one-time e-mail customer product
survey. What did Mandy do wrong?

Correct

Good choice. She collected too much personal information about customers.

Mandy used a database to collect personal information that might come in handy for future product
launches. Companies that collect information about consumers just in case risk annoying potential
and existing customers. Recipients of requests for information may feel that the company is taking
advantage of them while not providing anything of value in return.

Managing Upward:-
Question 1

You're having lunch one day with Kathy, a coworker from another department, when she says, "I feel
like my manager and I just aren't working toward the same goals lately." You suggest she try managing
upward to ...

IncorrectNot the correct choice. You answered: Convince Kathys manager to work toward the same
goals as Kathy.
Managing upward is not about convincing your boss to work toward your goals.

The correct choice is: Improve communication and increase Kathys effectiveness.

Managing upward is a process of influencing your manager to make decisions that benefit both of you.
Managing upward fosters open communication and helps build a strong relationship between you and
your manager, thereby increasing your effectiveness.

REVIEW MATERIAL

Learn > The Purpose of Managing Upward > What is managing upward?

Question 2

Which of the following can be considered a sign that your relationship with your manager has become
weak?

CorrectGood choice. You answered: She begins directing people who report to you.

When your manager begins providing direction to those who report to you, it is a sign that she feels
forced to manage around you.

REVIEW MATERIAL

Learn > The Purpose of Managing Upward > What if I don't manage upward?

Question 3

David has recently taken over as manager of your team. You want to drive the process of shaping your
relationship with him, so you setup a meeting to discuss mutual expectations. Which of the following
questions should you avoid during your meeting with David?

CorrectGood choice. You answered: I like to provide frequent updates and seek advice. How about I
contact you twice weekly?

Use your manager's time wisely. It is a good idea to avoid comments that suggest you may take up
Davids time or resources with trivial matters.

REVIEW MATERIAL

Learn > Developing a Relationship with Your Manager > How you can drive the process

Question 4

You are meeting with your manager to discuss the deadlines of your next project. You are concerned
that you will not meet the deadlines without assistance from a business expert from the client group to
define the content requirements. What should you do?

CorrectGood choice. You answered: Let your manager know that you will need support from a business
expert in the client group in order to meet the deadlines.

It is important to let your manager know what your expectations are of her. Advising, This is what I
need from you to be effective on this project is a great way to build a productive working relationship.

REVIEW MATERIAL

Learn > Developing a Relationship with Your Manager > Manage expectations
Question 5

You are the manager of a team of unionized technicians at an electronics manufacturing plant. One of
your supervisors corporate goals for this quarter is to achieve a record of zero accident reports within
his unit, in line with the organizations focus on increasing the health and safety of their employees.
You consider this an ambitious objective. What is your best course of action?

CorrectGood choice. You answered: Ask a representative from the health and safety department to
conduct a series of seminars with your team.

Reviewing proper health and safety procedures within your team is a great way to promote both your
supervisors and organizations goals.

REVIEW MATERIAL

Learn > Developing a Relationship with Your Manager > Promote your manager's goals

Question 6

You are a check-in agent at a major airline. Your manager, Steve, is working to increase operational
efficiency. You prepare a report detailing a process improvement that could result in a 15 percent
increase in efficiency and set up a meeting to discuss the idea. You e-mail the report to Steve a week
in advance. During the meeting, you discover that Steve appears not to have looked at your report and
asks you to explain the findings. What most likely went wrong?

CorrectGood choice. You answered: You should have determined whether Steve is a listener or a
reader.

Determining Steves communication style may have saved the misunderstanding. If Steve is a listener,
he likes to hear information first and read about it later.

REVIEW MATERIAL

Learn > Communicating with Your Manager > Understand your manager's communication style

Question 7

As team leader of quality management, you are meeting with your manager to discuss priorities. She
indicates that her top priorities are implementing a new reporting process, closing all problem
detection reports (PDRs) as soon as possible and rewarding employees who submitted successful
improvement suggestions last quarter. You negotiate the required resources and close the meeting. A
week later, your manager asks whether you have issued all of the employee rewards. She is
disappointed to hear you focused on closing the PDRs first. What could you have done differently?

CorrectGood choice. You answered: Asked for more specific language when discussing deadlines.

Your managers use of the term as soon as possible suggested that closing the PDRs was the highest
priority. When discussing deadlines, use specific language. Specifying dates for accomplishing each
goal would have cleared up any misunderstanding around prioritization.

REVIEW MATERIAL

Learn > Communicating with Your Manager > Use communication to boost productivity

Question 8
You are a team leader at the data processing center for a large financial institution. Your manager has
decided to cut costs by using a different supplier for the data storage tapes used in daily operations.
You disagree with this approach because you know the company has had previous customer service
issues with the proposed supplier, Mobius Data. What would be an appropriate response?

CorrectGood choice. You answered: Cutting costs is critical, but weve had bad experiences with
Mobius Data in the past. Acme Supply offers lower prices and has received excellent service reports.

The key is to disagree in a constructive manner. In this response, you reflect your managers concern
for cutting costs and offer an alternative supplier suggestion, rather than simply raising objections.

REVIEW MATERIAL

Learn > Communicating with Your Manager > When you disagree with your manager

Question 9

You manage a small team of trainers for a local transportation company. Your manager asks you to
ensure that all drivers are trained on the new vehicle maintenance schedule by the end of the month.
She explains the company is losing too much money to costly repairs that could be avoided by
following this new schedule. You understand the importance of the request, but your resources are
already committed to training a large group of new hires and implementing a high-priority, Web-based
training course. How should you respond?

CorrectGood choice. You answered: All resources are committed to the new hire training and Web-
based courses for the next three months. Would you review our scheduling priorities with me or
consider bringing in an additional resource on a short-term contract?

Working together with your manager to negotiate priorities is best achieved when you are clear about
the time requirements for all your projects. Ask your manager to review scheduling priorities or provide
alternatives if your priorities do not match.

REVIEW MATERIAL

Learn > Negotiating with Your Manager > Identify priorities

Question 10

As the manager of Sunrise Fitness Club, you have been trying to convince your manager to authorize
the purchase of an expensive new line of equipment for your club. Although you've presented your
proposal in detail, your manager is still not convinced that the money will be well spent. Which of the
following would be an effective response?

CorrectGood choice. You answered: The new equipment line can help us increase our membership,
since no other club in town offers it yet.Focusing on a win-win approach when negotiating is a good
strategy. Increased membership brings profitability that benefits both you and your manager.

Negotiating:-
Question 1

A consulting firm plans to buy office equipment from a retailer that is going out of business. The firm
wants to negotiate as low a price as possible; the retailer wants as high a price as possible. This is an
example of a ...
CorrectGood choice. You answered: Distributive negotiation.

This is distributive, or zero-sum, negotiation. The parties are competing over the distribution of the
benefits of an agreement. Often, there is only one issue in zero-sum negotiation: money.

REVIEW MATERIAL

Learn > Types of Negotiation > Distributive negotiation

Question 2

Which situation is not likely to involve integrative negotiation?

CorrectGood choice. You answered: An art dealer negotiating the price of a rare painting.

An art dealer negotiating the price of a painting is distributive negotiation, not integrative negotiation.
The art dealer is competing with the seller over the distribution of benefits (money). In win-win
negotiation, parties cooperate to achieve maximum benefits by integrating their interests into an
agreement.

REVIEW MATERIAL

Learn > Types of Negotiation > Integrative negotiation

Question 3

After more than 100 years in business, Global Furniture is selling out to a rival corporation. The deal
will involve a very small inventory of raw materials, but the price of the business will likely be very
high. The owner decides to conduct a multiphase negotiation. He hires a consultant to advise him.
Which of the following statements made by the consultant would be bad advice?

IncorrectNot the correct choice. You answered: You are most likely to walk away from the deal in the
first phase of negotiation.

This is actually good advice. If the negotiations become disconcerting, you should end the negotiations
in the first phase while you still cans.

The correct choice is: Deal with the easy issues such as valuation of the inventory first.

The consultant is unlikely to suggest this since it is bad advice. In multiphase negotiations, you should
ensure that the last phase is not the most significant in dollars or impact. Therefore, you would not
deal with the bigger issues in the last phase of the negotiation.

REVIEW MATERIAL

Learn > Multiphase and Multiparty Negotiations > Strategies for multiphase negotiations

Question 4

High Tech Inc. has negotiated a three-year contract with New Style Advertising. After the first year,
High Tech decides that it is unhappy with the creativity and flair in the advertising campaigns. How
could High Tech have negotiated differently to avoid being locked into an unsatisfactory three-year
relationship with New Style? High Tech could have ...
CorrectGood choice. You answered: Structured a multiphase transaction.

High Tech could have avoided this situation by structuring a multiphase transaction. Multiphase
transactions are negotiations that are implemented in phases or that have the prospect of future
dealings. Early phases allow parties to build trust by performing as promised. If this were a multiphase
transaction, High Techs dissatisfaction with New Styles work in an early phase might have signaled a
need to revise the contract terms or discontinue the relationship.

REVIEW MATERIAL

Learn > Multiphase and Multiparty Negotiations > Strategies for multiphase negotiations

Question 5

Jack is considering a management position in an accounting firm. If Jack cant negotiate a salary of at
least $90,000, he plans to keep his current job. The new firm cannot pay more than $95,000. If an
agreement cannot be reached, the accounting firm will have to hire a far less qualified candidate.
Which statement about Jacks BATNA (Best Alternative to a Negotiated Agreement) is true?

CorrectGood choice. You answered: Jacks BATNA is keeping his current job.

Jacks BATNA (Best Alternative to a Negotiated Agreement) is keeping his current job. BATNA means
knowing what you will do or what will happen if you do not reach agreement in negotiation.

REVIEW MATERIAL

Learn > Four Key Concepts in Negotiation > Best alternative to a negotiated agreement

Question 6

Alias Shipping negotiates the purchase of trucks with a supplier of delivery vehicles. Alias sets its
reservation price at $1,750,000. The supplier will not sell the vehicles for less than $2,250,000. What is
the ZOPA?

CorrectGood choice. You answered: There is no ZOPA.

There is no ZOPA. The ZOPA (Zone of Possible Agreement) is the area in which a deal can take place.
Each partys reservation price determines one end of the ZOPA. The ZOPA is the overlap between the
parties reservation prices. In this instance, the buyers reservation price is $1,750,000; the sellers is
$2,250,000. Thus, there is no overlapping range in which they could agree no ZOPA.

REVIEW MATERIAL

Learn > Four Key Concepts in Negotiation > Zone of possible agreement

Question 7

Youre grading an assignment for the business studies class youve been teaching. One of your
students clearly has not been paying attention to your lecture on negotiation because his essay is full
of errors. However, the student got one thing right. Which of the following sentences from the essay on
negotiation strategies is correct?

CorrectGood choice. You answered: "Choosing the right location for negotiations can affect the
outcome in your favor."

Logistical factors such as where the meeting occurs, who attends, and the sequence of the agenda can
influence the way others participate and react in meetings. If you feel that the meetings setting and
attendees have been rigged by the other party to produce a particular result that is not in your favor,
heed those feelings. Its likely that whoever set the agenda and called the meeting did so with a
particular outcome in mind.

REVIEW MATERIAL

Learn > Nine Steps to a Deal > Steps 7-9

Question 8

Lisa is negotiating to buy a new boat and trailer from Big Als Marine. Which of the following is the best
tactic for her to use? Lisa should ...

CorrectGood choice. You answered: Establish an anchor.

During distributive negotiations like buying a boat, it is a good idea to establish an anchor an offer
that sets the bargaining range. Its best to anchor when you have a strong sense of the other sides
reservation price; your proposal should be at or just a bit beyond that number. Dont be too aggressive
or greedy, or the other side may walk away.

REVIEW MATERIAL

Learn > Negotiation Tactics > Tactics for distributive negotiations

Question 9

The dissolution of the partnership Smith & Jones Automotive is fraught with high emotion. While
negotiating the dissolution, Smith realizes that Joness emotional needs have taken precedence over
logic. Smith understands that the source of Joness frustration is his loss of pride. He acknowledges his
partners feelings, expresses empathy, and offers reassurance. But the angry outbursts continue. What
is the best strategy for Smith at this point?

IncorrectNot the correct choice. You answered: Recognize Jones as a potential saboteur.

There is no indication of a saboteur at work here. Saboteurs are smaller internal or external
stakeholders (particularly in multiparty negotiations) who may have the power to block or sabotage a
good deal.

The correct choice is: Arrange to negotiate with a neutral facilitator.

The best strategy for Smith would be to arrange to negotiate with a neutral facilitator or with a
negotiator who is less emotional. Another good strategy is to call for a temporary break in
negotiations.

REVIEW MATERIAL

Learn > Barriers to Agreement > Difficulties in communication

Question 10

You are locked in a difficult negotiation with a die-hard bargainer. You havent let the other party
unnerve you with the insults and jabs. Still, the negotiation is not progressing, and youre unable to
move closer to agreement. What is your best strategy?

IncorrectNot the correct choice. You answered: Take a cooperative stance by being less guarded in the
information you disclose.
It is unwise to disclose too much information to die-hard bargainers. They may use it to exploit you.

The correct choice is: Suggest alternative options to see what that reveals.

Your best strategy would be to suggest some alternative packages or options and ask what the other
party prefers and why. Even if the party wont disclose any interests, you can learn something about
any preferences the options the party likes or dislikes. Knowing more about any preferences might
help you move closer to agreement.

REVIEW MATERIAL

Learn > Barriers to Agreement > Die-hard bargainers

Question 11

Juanita is the lead negotiator on a team that is handling the sale of a mineral extraction company. She
knows her weakness as a negotiator and is determined to overcome it. Before starting, she works with
her team to set a reasonable reservation price. She also reviews her BATNA and schedules breaks
every 20 minutes during the negotiations. What mental error is she actively trying to avoid?

CorrectGood choice. You answered: Irrational escalation.

The best way to avoid irrational escalation is to know your BATNA before you negotiate, work with your
team ahead of time to set a reasonable reservation price, and set clear breakpoints to assess your
situation to ensure you arent getting off track.

REVIEW MATERIAL

Learn > Mental Errors > Irrational escalation

Question 12

Your favorite business magazine has a cover story this month about Mancipo, a young businessman
who is quickly earning a reputation as one of the best negotiators around. Which of the following
excerpts is most likely to appear in the article?

CorrectGood choice. You answered: "'Sometimes the key to good negotiating is helping the other side
figure out what they want,' Mancipo says."

A good negotiator identifies the interests of both sides and develops value-creating options. He or she
can help the other side see the value of sharing information and expanding value opportunities,
particularly in integrative negotiations.
Business Case:-
Question 1

Your coworker Samantha is a recent graduate just starting her first job. Shes still trying to get used to
the office jargon and is especially confused about the difference between a business plan and a
business case. You decide that the best way to explain it is to give an example of when she would
create a business case. Which of the following examples would you be least likely to give?

CorrectGood choice. You answered: To lay out how your business unit will integrate a recently acquired
company.

A business plan would be more appropriate for laying out how your business unit will integrate a
recently acquired company. Business plans generally feature long-range projections of revenues,
expenses, and other information to secure financing from investors or to plan strategy execution.

REVIEW MATERIAL

Learn > Business Case Basics > Business case vs. business plan

Question 2

You want to develop a business case for considering whether to combine the copywriting and technical
writing departments under a single manager. Which of the following would not be part of the early
stage of building your business case?

Correct

Good choice. You answered: Explore the potential risks youd encounter if you combined the
departments.

While identifying potential risks is important, you would not do so at the early stage of building your
business case. It is more important at this stage to define the opportunity, identify the most relevant
business objectives, and measure solutions against the metrics considered important by your
company.

REVIEW MATERIAL

Learn > Business Case Basics > How to build a business case

Question 3

Billedeau Products Inc. manufactures low-priced bathroom fixtures. Competition in that segment of the
market is increasing and sales are down. Cindy, the vice president of sales, might have to lay off sales
staff to make up for falling revenues. However, she has noticed opportunities in the high end of the
market. She thinks that the company should migrate its product lines to higher quality items with
bigger profit margins. Cindy decides to write an opportunity statement for the idea. Which of the
following is the best opportunity statement?

Incorrect
Not the correct choice. You answered: There are many ways to increase profitability at Billedeau
Products but by far the most effective is to move into a different market segment.

When drafting an opportunity statement, avoid the common mistake of defining the opportunity by
describing your preferred solution.

The correct choice is: Billedeau Products will become more competitive by identifying the best
solution for increasing revenues and profits.

A good opportunity statement describes the benefits that will come with solving the problem or seizing
the opportunity. In other words, answer the question, How will my group/unit/company benefit from
spending resources to address this issue?

REVIEW MATERIAL

Learn > Defining the Opportunity > Develop an opportunity statement

Question 4

You are mentoring a colleague on how to create a business case. You tell her that one of the initial
steps is to identify alternatives. When you review her list of alternatives, you note that she has made a
common mistake. Which of the following is a common pitfall during this stage?

Correct

Good choice. You answered: Exploring only the solutions for which she has a strong preference.

It is a pitfall if you have a strong preference for a particular solution at the outset and therefore fail to
explore other alternatives.

REVIEW MATERIAL

Learn > Identifying Alternatives > Generate a list of alternatives

Question 5

Lars is writing a business case for whether to outsource a project to a contractor or to do the work in-
house. He has already identified the opportunity and listed the potential alternatives. He is now in the
process of gathering and analyzing data for each alternative, and asks for your advice. You respond
that at this stage he should:

Incorrect

Not the correct choice. You answered: Increase credibility by gathering information only from experts
with expertise in the choices he is considering.

It is a good idea to consult several sources for each piece of information, including sources such as the
Internet, colleagues, and friends outside of your organization.

The correct choice is: Obtain the available historical data to help him assess each alternative.

Wherever possible, you should start by obtaining historical and/or typical data. Youll then want to
forecast changes in that data that would result from each of the alternatives youre comparing.

REVIEW MATERIAL

Learn > Gathering Data and Estimating a Time Frame > Gather the data
Question 6

You have put John, your direct report, in charge of building a business case for changes to the
companys environmental practices. He presents you with a list of alternative courses of action, but
you quickly realize he has presented too many alternatives. You want to help him avoid this mistake in
the future. What advice would you be least likely to give John about how to narrow the list of
alternatives?

Correct

Good choice. You answered: During brainstorming sessions, accept or reject suggestions as they come
up.

The brainstorming stage is not the time to narrow down your options. Record the results of a
brainstorming session on a flip chart or whiteboard, without judging them or discussing their potential
pros and cons. Your goal at that stage is to generate as many feasible alternatives as possible.

REVIEW MATERIAL

Learn > Identifying Alternatives > Narrow your choices

Question 7

Brandy is preparing a business case for whether to replace the computers and printers in her
department. She is trying to analyze the information she has collected prior to making a decision. She
asks you for advice. What advice would you be least likely to give her?

Incorrect

Not the correct choice. You answered: Make a series of assumptions founded on your information.

It is important to make a series of assumptions about the impact of each alternative. This will help you
to choose the most appropriate solution for your business case.

The correct choice is: At this stage, dont include the status quo among your alternatives.

Failing to consider the status quo (the current condition) as an alternative is a common pitfall in
preparing a business case.

REVIEW MATERIAL

Learn > Identifying Alternatives > Narrow your choices

Question 8

You and some of your colleagues are having coffee together. The discussion has turned to ways of
choosing a solution for a business case that you are considering. Everyone has some ideas. Whose
advice would you be least likely to take?

Correct

Good choice. You answered: Ramn said, You have good instincts. Stick with your initial
recommendation.
While you may need to rely on your intuition and best judgment in the selection process, your initial
recommendation may not be the best choice. You should be willing to modify or even abandon it if your
analysis indicates the option is too risky for you or for the organization.

REVIEW MATERIAL

Learn > Choosing a Solution and Assessing Risks > Select the best solution

Question 9

You have constructed a high-level implementation plan for your business case and will be presenting it
to upper management in two weeks. You want to include key pieces of information that are important
to the decision makers. Which item in the following list would not be part of the presentation?

Correct

Good choice. You answered: Detailed information on how each milestone will be accomplished.

Decision makers will want to understand each of your milestones to ensure that your project is
feasible, but they do not need the details of how you will accomplish each one.

REVIEW MATERIAL

Learn > Creating a High-Level Implementation Plan > Check in with decision makers

Question 10

Judy has been asked to make a formal presentation of a business case to upper management at their
monthly meeting. Shes very nervous because it is her first time in front of this group. She phones one
of her favorite professors at the university where she studied business and asks him for some advice
on making an effective presentation. What tip is he most likely to give her?

Correct

Good choice.You answered: Create a visual presentation of your case, even if company policy only
requires a written document.

Consider creating a visual presentation comprising five to seven slides, even if your company doesnt
require it. These slides would correspond approximately to the steps you followed to build your
business case.
Customer Focus:-

Question 1

Masie Cheng, product manager at Ascot Systems, has to prepare a marketing plan for her
department. The company provides accounting systems and related services to small and
mid size companies. Masie wants to develop a plan that will improve long-term profitability.
Which one of these strategies should Masie propose?

Correct

Good choice. You answered: Develop a value-added newsletter and a referral incentive
program for current customers.

Masie's plan should develop relationships with long-term customers. Instead of focusing
nearly all the company's energy and money on getting new customers, she should look for
ways to satisfy and reward the loyalty of her current customers. Studies show that the
longer customers are loyal, the more profitable they become.

REVIEW MATERIAL
Learn > Facts about Customers and Loyalty > The three Rs

Question 2

You've been asked to comment on your department's proposed marketing budget. You see
that the marketing committee has proposed spending 85 percent on acquiring new
customers and 15 percent on customer retention. What would you recommend to the
marketing committee?
Correct

Good choice. You answered: Increase the amount on current customers, because current
customers are more profitable.

It is current customers who represent the greatest profit potential. Money spent on them is
money well spent. The proposed budget is based on the misconception that if you want to
make a profit, you must increase market share and that any customer is a good customer.

REVIEW MATERIAL
Learn > Facts about Customers and Loyalty > Misguided marketing

Question 3

Which of the following statements about customer loyalty is not accurate?

Correct

Good choice. You answered: Loyal customers demand lower pricing than new customers.

Loyal customers are often less sensitive to price than new customers.

REVIEW MATERIAL
Learn > Facts about Customers and Loyalty > The three Rs

Question 4

A new marketing whiz kid is contracted to revamp your departments marketing effort.
After studying your department and its customers, the consultant issues his report. Which of
the following statements is most likely in that report?

Correct

Good choice. You answered: Increasing market share may attract customers who are not
profitable.

There is a misconception that if you want to make a profit, you must increase market share.
Simply trying to increase the number of new customers may not be enough because some
customers are completely wrong for your company.
REVIEW MATERIAL
Learn > Facts about Customers and Loyalty > Misguided marketing

Question 5

What is likely to occur three to four years into a customer relationship with your company?
Your customers ...

Correct

Good choice. You answered: Become less concerned with price and cheaper to serve.

Research shows that the customer becomes less price-sensitive when making purchases
after initial product or service purchases. Having become knowledgeable about the company
and its policies, the customer is less expensive to serve and therefore costs are reduced.

REVIEW MATERIAL
Learn > How Loyalty Affects Profitability > Lifetime value

Question 6

Akiko, marketing manager at HEJ Co. Ltd., wants to know how much current customers
contribute to profit growth. She runs several analyses on customer sales data and finds

Correct

Good choice. You answered: Profitability increases from year to year throughout the
relationship.

Research on service industries shows that relationships with typical customers grow
increasingly more profitable over time regardless of the industry. Profit growth comes from
increased purchases, lower service and operating costs, and referrals.

REVIEW MATERIAL
Learn > How Loyalty Affects Profitability > Lifetime value

Question 7

The Whiteoaks Garden Center has a stable staff and a reputation for excellent customer
satisfaction. However, a hiring freeze followed by staff layoffs over the past few months has
put increasing pressures on the floor staff. Around the same time, customer satisfaction
ratings have begun to slide substantially and the company is now worried about customer
defection. Can you identify the most likely source of the problem in the Service Profit Chain?

Correct

Good choice. You answered: Employee satisfaction.

Whiteoaks should look to employee satisfaction. The hiring freeze and layoffs mean more
work for surviving employees and could most likely contribute to lower employee
satisfaction. When employees enjoy their work and believe they are making a difference,
they tend to stay longer and to become more productive and knowledgeable, which in turn
creates greater customer satisfaction.

REVIEW MATERIAL
Learn > How Loyalty Affects Profitability > The service-profit chain

Question 8

As a result of a recent survey, Home Source, a large "do-it-yourself" retail chain, learns that
customer satisfaction is low. To solve the problem, management considers where to focus
customer service training the cashiers, sales associates, yard associates, or management.
The decision is made to invest in the training of sales associates. Why might this approach
have limited effectiveness?

Correct

Good choice. You answered: Everyone in the organization needs to be customer focused.

Employee capability cannot stop at the front line. Many organizations understand the need
for training frontline workers to be polite, empathetic, and knowledgeable. But great
frontline service is not enough. Home Source needs to have a plan that makes everyone in
the company responsible for delivering the results the customer wants.

REVIEW MATERIAL
Learn > Building Employee Capability > Go beyond frontline service

Question 9

Best Rate Inc., an insurance company, has seen considerable growth in its telephone-based
direct sales division. The management team has developed several strategies to meet its
staffing requirements and add additional telephone sales agents. What would be the best
strategy? Should they ...
Correct

Good choice. You answered: Hire people with the right attitude.

Best Rate has a chance to create a cycle of success by starting at the beginning. They can
pursue the approach of most successful service organizations: hire first for attitude and only
secondly for skills. Then, train new hires in the interpersonal and technical skills they need
for their jobs.

REVIEW MATERIAL
Learn > Building Employee Capability > Select for attitude, train for skills

Question 10

The Frostee Kream ice-cream store chain has a problem. Employee turnover is high,
negatively impacting morale, customer satisfaction, sales, and profits. The human resources
manager develops a four-part strategy to break the cycle of mediocrity and start a cycle of
success. As CEO of the company, you review the four strategies and discover one strategy
that will not work. Which of the strategies would you advise against?

Correct

Good choice. You answered: Encourage managers to review and direct employees
throughout the workday.

Micromanagement will not stop employee turnover it is demeaning and frustrating to


capable employees. They resent being treated as incompetent, and they become
dissatisfied with their work. The most valuable employees will find work somewhere else.

REVIEW MATERIAL
Learn > Building Employee Capability > Allow latitude within limits

Question 11

Modesto Drug and Beauty, a pharmacy and cosmetics chain, has begun using mystery
shopping to audit the customer service experience at stores that have weak sales and low
customer satisfaction. Managers used the results of the secret shopping to fire one
employee and give formal reprimands to two others. However, sales and customer
satisfaction have not improved. What could be wrong with this approach to improving
customer service?
Incorrect

Not the correct choice. You answered: Mystery shopping is not an objective method for
gathering customer service information.

Mystery shopping is thought to offer a high degree of objectivity.

The correct choice is: Information on customer service should not be used to level
criticism.

All tools that measure feedback, including mystery shopping, are best used in an
atmosphere of trust. They should be used to gather useful information as a way to improve
products and services, not as weapons or methods for blaming or punishing people.
Modesto's continuing problems could stem from low employee satisfaction since employees
can regard mystery shopping as unfair or even as "spying."

REVIEW MATERIAL
Learn > Knowing the Customer > Tools for getting feedback

Question 12

Crowfoot Unlimited, a precision machinery firm, is developing a group of new products. A


series of internal meetings reveals that the company is unfocused and confused about its
direction. The senior sales engineer suggests looking at what the customers think. So a
management team draws up a number of possible approaches to audit customers opinions.
Which approach should Crowfoot avoid?

Incorrect

Not the correct choice. You answered: Follow-up satisfaction calls.

This could be an appropriate approach. Many companies have established a follow-up


satisfaction call as another listening post in their customer relationships. More elaborate
than a card, but less involved than a comprehensive survey, the follow-up call takes place
shortly after a transaction.

The correct choice is: Focus groups.

Focus groups are excellent for testing products and services, but the focus group
phenomenon can send an unfocused company into disarray as it becomes unable to think
clearly in the face of often-contradictory results.
REVIEW MATERIAL
Learn > Knowing the Customer > Tools for getting feedback

Question 13

At Light Source Outlets, which recently moved to less expensive quarters further from key
shopping areas, prices have increased slightly over the last quarter. However, the results
and process quality of its goods have remained unchanged. How would this impact the
customer value according to the Customer Value Equation?

Correct

Good choice. You answered: Decreases customer value.

Prices and access costs, which are in the denominator of the Customer Value Equation, have
gone up, while results and process quality, which are in the numerator, have not. The result
is a decrease in customer value.

REVIEW MATERIAL
Learn > Delivering Value > Customer Value Equation

Question 14

Luis Rosario, account services manager at the print service bureau Multi-Print, is working on
a set of satisfaction goals for profitable customers. He's reviewing the draft list and identifies
one that is not an appropriate customer satisfaction goal for the company to pursue. Which
one is not a good satisfaction goal?

Incorrect

Not the correct choice. You answered: We will always ask our customers if they got
everything they needed at each visit.

This is a good customer satisfaction goal. Listening to customers relates to attitude and
communication, both important customer service factors.

The correct choice is: We will strive to have every customer become a repeat customer.

Luis realizes that having every customer become a repeat customer is not a good customer
satisfaction goal for the company, because it is not in Multi-Prints interest to pursue
customers who will not be profitable. He knows that satisfaction goals should focus on the
companys most profitable customers. Some customers will never be satisfied and will
therefore not be profitable.

REVIEW MATERIAL
Learn > Delivering Value > Develop satisfaction goals for profitable customers

Question 15

Which of the following statements about improving service recovery is false?

Incorrect

Not the correct choice. You answered: Recovering from mistakes can increase customer
loyalty and profitability.
This is true. Research shows that recovering from mistakes can actually increase customer
loyalty and contribute to profitability.

The correct choice is: Employees must be provided with explicit problem-solving
directions.

It is false that employees need explicit problem-solving directions. Employees must be given
latitude to solve problems. Without proper latitude, an organization cannot execute recovery
procedures effectively.

REVIEW MATERIAL
Learn > Delivering Value > Improve service recovery

VIRTUAL TEAMS
Question 1

You are the author of Not Quite All There, a business book about virtual teams. A
junior editor has written a promotional paragraph that will appear on the back cover
of the book and has asked you to review it. You quickly realize that the editor did
not read your book very carefully because she got only one sentence right. Which
sentence is correct?

CorrectGood choice. You answered: One of the biggest potential advantages of


virtual teams is their diversity.

Virtual teams make it easier for organizations to bring together an extremely


diverse group of people with varying skills, experiences, and knowledge about
customers and business.
REVIEW MATERIAL

Learn > Working with a Virtual Team > The benefits of a virtual team

Question 2

Kasha is a project lead for a team of management consultants who will be involved
with the restructuring of a multinational firm. At the team kickoff, Kasha describes
her views for working with a virtual team. Her team members, all experts in their
respective fields, identify one of her statements as not good practice. Which
statement is it?

IncorrectNot the correct choice. You answered: Well define all roles at the
beginning to ensure that there is no redundancy or confusion.

This is good practice. A virtual team stands little chance of success if team
members roles are not clearly defined from the beginning.

The correct choice is: My role is to provide strong central leadership to keep this
team pulled together.

On virtual teams, strong central leadership is not good practice ideally,


leadership should shift depending upon the task at hand. As on any team in which
many highly competent people are brought together, leadership must be shared
because no one team member has all the answers. Leaders and coordinators should
be willing to share leadership, while team members should be willing to contribute
to it.

REVIEW MATERIAL

Learn > Establishing a Foundation for Success > Share leadership

Question 3

Leo wants to recruit a virtual team of vendors that he can rely on to launch his
company's new software products globally. When he discusses his plan with his
director, the director cautions him to do a "culture check." What does his director
mean?

CorrectGood choice. You answered: Inquire how team members view working
conditions and etiquette.

A culture check is a discussion with your virtual team to learn how they feel about
working conditions, hours, authority and delegation, communication (what is
considered polite or not), and other culturally determined activities. Since team
members views on these issues may differ depending on where they live, these
activities should be carefully considered and discussed openly.

REVIEW MATERIAL

Learn > Establishing a Foundation for Success > Perform a "culture check"

Question 4

Your colleague, Evelyn, confides in you that her product design team residing in
New York, London, and Frankfurt is not meeting her expectations. She asks you
what to do to build community and mold this group of strangers into a team. What
is the best advice you can give her?

CorrectGood choice. You answered: Build trust among team members: Set time
aside for just getting to know one another.

Trust is perhaps the most important and the most difficult quality for any high-
performing team to achieve. To build trust quickly, set aside time for people to
socialize and get to know one another.

REVIEW MATERIAL

Learn > Fostering Team Identity and Collaboration > Set up times for people to talk
to each other

Question 5

Four interns are discussing the best way to exchange information within the virtual
team theyve joined. Which one is right?

CorrectGood choice. You answered: Chieu says, Leave it up to the team leader to
decide. Thats her job.

Leaders of virtual teams should decide who will submit what kinds of information to
whom, and how often they will do it. The challenge is to ensure that team members
communicate progress on their parts of the project, and that the resulting
information and updates circulate easily among all team members.

REVIEW MATERIAL

Learn > Aligning Team Members through Communication > Keep everyone informed

Question 6

Until recently, Kurt has always worked in an office where he has enjoyed the
collegiality of his coworkers and being part of a close-knit team. Now, after a
company merger, Kurt has been assigned to a virtual team and must work from
home. As Kurt's team leader, you notice that his morale is low. How do you coach
him?

CorrectGood choice. You answered: Suggest that Kurt get out of the house each day
to meet associates or friends.

Kurt is experiencing isolation and needs to get out of the house every day. Isolation
is often a problem when people accustomed to working in an office with colleagues
suddenly find themselves spending a lot of time alone. Coaching involves
encouraging the team member to obtain the social contact he needs. If the team
member works at home, it is recommended that he get out of his home office every
day go out for lunch, meet associates, visit friends, or even go for a walk.

REVIEW MATERIAL

Learn > Coaching a Virtual Team > Clarify priorities and build team identity

Question 7

You're an information technology consultant. The leader of a virtual team has hired
you for a fixed fee to advise her on the technology needs of her team. Which of the
following would be the best advice to give her?

CorrectGood choice. You answered: Before deciding, track the expected flow of
information, so that you can see what information you will need and where you will
need it.

Team leaders must assess technology needs and help their teams implement
different types of technologies. Tracking the expected flow of information is helpful
in designing an information system that meets your teams needs.

REVIEW MATERIAL

Learn > Making Technology Decisions > Assess technology needs

Question 8

At the start of a project, Kate prepares some e-mail guidelines for her virtual team.
Which of the following policies would you advise against?

CorrectGood choice. You answered: Copy (cc) everyone on your immediate team in
your e-mails.

There is probably no need to copy every person on your team list with every
correspondence you send out. Nobody wants to receive massive amounts of e-mail.
Ensure that every e-mail you send is relevant to each recipient.

REVIEW MATERIAL
Learn > Making Technology Decisions > E-mail

Question 9

Rosa is the technology expert on her team responsible for choosing and setting up
the electronic communications for her group. Rosa has several technology
arrangements and options to choose from. Her team lead suggests that Rosa look
into using a "four-walled model." What it is this?

CorrectGood choice. You answered: A virtual team room with areas for four key
types of information.

The four-walled model is a way of setting up a project Web site to create a virtual
team room. The four walls are: purpose (an area of the site where goals, tasks,
and deliverables are posted); people (an area where team members and roles are
defined); documents (an area where meeting schedules, agendas, minutes, and
presentations are stored, and current documents in progress are posted for
comment); and communications (an area containing links and information that
connect everyone on the team).

REVIEW MATERIAL

Learn > Making Technology Decisions > Web sites and intranet sites

Question 10

Which of the following technologies is the quickest and easiest way for virtual team
members to communicate verbally about a problem team members are
experiencing?

CorrectGood choice. You answered: Telephone conferencing.

Telephone conferencing, relative to the other technologies, is the quickest and


easiest way for virtual team members to communicate verbally.

REVIEW MATERIAL

Learn > Using Technology for Live Meetings > Telephone conferencing

Vous aimerez peut-être aussi